U.S. flag

An official website of the United States government

The .gov means it's official. Federal government websites often end in .gov or .mil. Before sharing sensitive information, make sure you're on a federal government site.

The site is secure. The https:// ensures that you are connecting to the official website and that any information you provide is encrypted and transmitted securely.

  • Publications
  • Account settings
  • Browse Titles

NCBI Bookshelf. A service of the National Library of Medicine, National Institutes of Health.

StatPearls [Internet]. Treasure Island (FL): StatPearls Publishing; 2024 Jan-.

Cover of StatPearls

StatPearls [Internet].

Case study: 60-year-old female presenting with shortness of breath.

Deepa Rawat ; Sandeep Sharma .

Affiliations

Last Update: February 20, 2023 .

  • Case Presentation

The patient is a 60-year-old white female presenting to the emergency department with acute onset shortness of breath.  Symptoms began approximately 2 days before and had progressively worsened with no associated, aggravating, or relieving factors noted. She had similar symptoms approximately 1 year ago with an acute, chronic obstructive pulmonary disease (COPD) exacerbation requiring hospitalization. She uses BiPAP ventilatory support at night when sleeping and has requested to use this in the emergency department due to shortness of breath and wanting to sleep.

She denies fever, chills, cough, wheezing, sputum production, chest pain, palpitations, pressure, abdominal pain, abdominal distension, nausea, vomiting, and diarrhea.

She reports difficulty breathing at rest, forgetfulness, mild fatigue, feeling chilled, requiring blankets, increased urinary frequency, incontinence, and swelling in her bilateral lower extremities that are new-onset and worsening. Subsequently, she has not ambulated from bed for several days except to use the restroom due to feeling weak, fatigued, and short of breath.

There are no known ill contacts at home. Her family history includes significant heart disease and prostate malignancy in her father. Social history is positive for smoking tobacco use at 30 pack years. She quit smoking 2 years ago due to increasing shortness of breath. She denies all alcohol and illegal drug use. There are no known foods, drugs, or environmental allergies.

Past medical history is significant for coronary artery disease, myocardial infarction, COPD, hypertension, hyperlipidemia, hypothyroidism, diabetes mellitus, peripheral vascular disease, tobacco usage, and obesity.  Past surgical history is significant for an appendectomy, cardiac catheterization with stent placement, hysterectomy, and nephrectomy.

Her current medications include fluticasone-vilanterol 100-25 mcg inhaled daily, hydralazine 50 mg by mouth, 3 times per day, hydrochlorothiazide 25 mg by mouth daily, albuterol-ipratropium inhaled every 4 hours PRN, levothyroxine 175 mcg by mouth daily, metformin 500 mg by mouth twice per day, nebivolol 5 mg by mouth daily, aspirin 81 mg by mouth daily, vitamin D3 1000 units by mouth daily, clopidogrel 75 mg by mouth daily, isosorbide mononitrate 60 mg by mouth daily, and rosuvastatin 40 mg by mouth daily.

Physical Exam

Initial physical exam reveals temperature 97.3 F, heart rate 74 bpm, respiratory rate 24, BP 104/54, HT 160 cm, WT 100 kg, BMI 39.1, and O2 saturation 90% on room air.

Constitutional:  Extremely obese, acutely ill-appearing female. Well-developed and well-nourished with BiPAP in place. Lying on a hospital stretcher under 3 blankets.

HEENT: 

  • Head: Normocephalic and atraumatic
  • Mouth: Moist mucous membranes 
  • Macroglossia
  • Eyes: Conjunctiva and EOM are normal. Pupils are equal, round, and reactive to light. No scleral icterus. Bilateral periorbital edema present.
  • Neck: Neck supple. No JVD present. No masses or surgical scarring. 
  • Throat: Patent and moist

Cardiovascular:  Normal rate, regular rhythm, and normal heart sound with no murmur. 2+ pitting edema bilateral lower extremities and strong pulses in all four extremities.

Pulmonary/Chest:  No respiratory status distress at this time, tachypnea present, (+) wheezing noted, bilateral rhonchi, decreased air movement bilaterally. The patient was barely able to finish a full sentence due to shortness of breath.

Abdominal:  Soft. Obese. Bowel sounds are normal. No distension and no tenderness

Skin: Skin is very dry

Neurologic: Alert, awake, able to protect her airway. Moving all extremities. No sensation losses

  • Initial Evaluation

Initial evaluation to elucidate the source of dyspnea was performed and included CBC to establish if an infectious or anemic source was present, CMP to review electrolyte balance and review renal function, and arterial blood gas to determine the PO2 for hypoxia and any major acid-base derangement, creatinine kinase and troponin I to evaluate the presence of myocardial infarct or rhabdomyolysis, brain natriuretic peptide, ECG, and chest x-ray. Considering that it is winter and influenza is endemic in the community, a rapid influenza assay was obtained as well.

Largely unremarkable and non-contributory to establish a diagnosis.

Showed creatinine elevation above baseline from 1.08 base to 1.81, indicating possible acute injury. EGFR at 28 is consistent with chronic renal disease. Calcium was elevated to 10.2. However, when corrected for albumin, this corrected to 9.8 mg/dL. Mild transaminitis is present as seen in alkaline phosphatase, AST, and ALT measurements which could be due to liver congestion from volume overload.

Initial arterial blood gas with pH 7.491, PCO2 27.6, PO2 53.6, HCO3 20.6, and oxygen saturation 90% on room air, indicating respiratory alkalosis with hypoxic respiratory features.

Creatinine kinase was elevated along with serial elevated troponin I studies. In the setting of her known chronic renal failure and acute injury indicated by the above creatinine value, a differential of rhabdomyolysis is determined.

Influenza A and B: Negative

Normal sinus rhythm with non-specific ST changes in inferior leads. Decreased voltage in leads I, III, aVR, aVL, aVF.

Chest X-ray

Findings: Bibasilar airspace disease that may represent alveolar edema. Cardiomegaly noted. Prominent interstitial markings were noted. Small bilateral pleural effusions

Radiologist Impression: Radiographic changes of congestive failure with bilateral pleural effusions greater on the left compared to the right

  • Differential Diagnosis
  • Acute on chronic COPD exacerbation
  • Acute on chronic renal failure
  • Bacterial pneumonia
  • Congestive heart failure
  • Pericardial effusion
  • Hypothyroidism
  • Influenza pneumonia
  • Pulmonary edema
  • Pulmonary embolism
  • Confirmatory Evaluation

On the second day of the admission patient’s shortness of breath was not improved, and she was more confused with difficulty arousing on conversation and examination. To further elucidate the etiology of her shortness of breath and confusion, the patient's husband provided further history. He revealed that she is poorly compliant with taking her medications. He reports that she “doesn’t see the need to take so many pills.”

Testing was performed to include TSH, free T4, BNP, repeated arterial blood gas, CT scan of the chest, and echocardiogram. TSH and free T4 evaluate hypothyroidism. BNP evaluates fluid load status and possible congestive heart failure. CT scan of the chest will look for anatomical abnormalities. An echocardiogram is used to evaluate left ventricular ejection fraction, right ventricular function, pulmonary artery pressure, valvular function, pericardial effusion, and any hypokinetic area.

  • TSH: 112.717 (H)
  • Free T4: 0.56 (L)
  • TSH and Free T4 values indicate severe primary hypothyroidism. 

BNP can be falsely low in obese patients due to the increased surface area. Additionally, adipose tissue has BNP receptors which augment the true BNP value. Also, African American patients with more excretion may have falsely low values secondary to greater excretion of BNP. This test is not that helpful in renal failure due to the chronic nature of fluid overload. This allows for desensitization of the cardiac tissues with a subsequent decrease in BNP release.

Repeat arterial blood gas on BiPAP ventilation shows pH 7.397, PCO2 35.3, PO2 72.4, HCO3 21.2, and oxygen saturation 90% on 2 L supplemental oxygen.

CT chest without contrast was primarily obtained to evaluate the left hemithorax, especially the retrocardiac area.

Radiologist Impression: Tiny bilateral pleural effusions. Pericardial effusion. Coronary artery calcification. Some left lung base atelectasis with minimal airspace disease.

Echocardiogram

The left ventricular systolic function is normal. The left ventricular cavity is borderline dilated.

The pericardial fluid is collected primarily posteriorly, laterally but not apically. There appeared to be a subtle, early hemodynamic effect of the pericardial fluid on the right-sided chambers by way of an early diastolic collapse of the RA/RV and delayed RV expansion until late diastole. A dedicated tamponade study was not performed. 

The estimated ejection fraction appears to be in the range of 66% to 70%. The left ventricular cavity is borderline dilated.

The aortic valve is abnormal in structure and exhibits sclerosis.

The mitral valve is abnormal in structure. Mild mitral annular calcification is present. There is bilateral thickening present. Trace mitral valve regurgitation is present.

  • Myxedema coma or severe hypothyroidism
  • Pericardial effusion secondary to myxedema coma
  • COPD exacerbation
  • Acute on chronic hypoxic respiratory failure
  • Acute respiratory alkalosis
  • Bilateral community-acquired pneumonia
  • Small bilateral pleural effusions
  • Acute mild rhabdomyolysis
  • Acute chronic, stage IV, renal failure
  • Elevated troponin I levels, likely secondary to Renal failure 
  • Diabetes mellitus type 2, non-insulin-dependent
  • Extreme obesity
  • Hepatic dysfunction

The patient was extremely ill and rapidly decompensating with multisystem organ failure, including respiratory failure, altered mental status, acute on chronic renal failure, and cardiac dysfunction. The primary concerns for the stability of the patient revolved around respiratory failure coupled with altered mental status. In the intensive care unit (ICU), she rapidly began to fail BiPAP therapy. Subsequently, the patient was emergently intubated in the ICU.  A systemic review of therapies and hospital course is as follows:

Considering the primary diagnosis of myxedema coma, early supplementation with thyroid hormone is essential. Healthcare providers followed the American Thyroid Association recommendations, which recommend giving combined T3 and T4 supplementation; however, T4 alone may also be used. T3 therapy is given as a bolus of 5 to 20 micrograms intravenously and continued at 2.5 to 10 micrograms every 8 hours. An intravenous loading dose of 300 to 600 micrograms of T4 is followed by a daily intravenous dose of 50 to 100 micrograms. Repeated monitoring of TSH and T4 should be performed every 1 to 2 days to evaluate the effect and to titrate the dose of medication. The goal is to improve mental function. Until coexistent adrenal insufficiency is ruled out using a random serum cortisol measurement, 50 to 100 mg every 8 hours of hydrocortisone should be administered. In this case, clinicians used hydrocortisone 100 mg IV every 8 hours. Dexamethasone 2 to 4 mg every 12 hours is an alternative therapy.

The patient’s mental status rapidly worsened despite therapy. In the setting of her hypothyroidism history, this may be myxedema coma or due to the involvement of another organ system. The thyroid supplementation medications and hydrocortisone were continued. A CT head without contrast was normal.

Respiratory

For worsening metabolic acidosis and airway protection, the patient was emergently intubated. Her airway was deemed high risk due to having a large tongue, short neck, and extreme obesity. As the patient’s heart was preload dependent secondary to pericardial effusion, a 1-liter normal saline bolus was started. Norepinephrine was started at a low dose for vasopressor support, and ketamine with low dose Propofol was used for sedation. Ketamine is a sympathomimetic medication and usually does not cause hypotension as all other sedatives do. The patient was ventilated with AC mode of ventilation, tidal volume of 6 ml/kg ideal body weight, flow 70, initial fio2 100 %, rate 26 per minute (to compensate for metabolic acidosis), PEEP of 8.

Cardiovascular

She was determined to be hemodynamically stable with a pericardial effusion. This patient’s cardiac dysfunction was diastolic in nature, as suggested by an ejection fraction of 66% to 70%. The finding of posterior pericardial effusion further supported this conclusion. The posterior nature of this effusion was not amenable to pericardiocentesis. As such, this patient was preload dependent and showed signs of hypotension. The need for crystalloid fluid resuscitation was balanced against the impact increased intravascular volume would have on congestive heart failure and fluid overload status. Thyroid hormone replacement as above should improve hypotension. However, vasopressor agents may be used to maintain vital organ perfusion targeting a mean arterial pressure of greater than 65 mm Hg as needed. BP improved after fluid bolus, and eventually, the norepinephrine was stopped. Serial echocardiograms were obtained to ensure that the patient did not develop tamponade physiology. Total CK was elevated, which was likely due to Hypothyroidism compounded with chronic renal disease.

Infectious Disease

Blood cultures, urine analysis, and sputum cultures were obtained. The patient's white blood cell count was normal. This is likely secondary to her being immunocompromised due to hypothyroidism and diabetes. In part, the pulmonary findings of diffuse edema and bilateral pleural effusions can be explained by cardiac dysfunction. Thoracentesis of pleural fluid was attempted, and the fluid was analyzed for cytology and gram staining to rule out infectious or malignant causes as both a therapeutic and diagnostic measure. Until these results return, broad-spectrum antibiotics are indicated and may be discontinued once the infection is ruled out completely.

Gastrointestinal

Nasogastric tube feedings were started on the patient after intubation. She tolerated feedings well. AST and ALT were mildly elevated, which was thought to be due to hypothyroidism, and as the TSH and free T4 improved, her AST and ALT improved. Eventually, these values became normal once her TSH level was close to 50.

Her baseline creatinine was found to be close to 1.08 in prior medical records. She presented with a creatinine of 1.8 in the emergency department. Since hypothyroidism causes fluid retention in part because thyroid hormone encourages excretion of free water and partly due to decreased lymphatic function in returning fluid to vascular circulation.  Aggressive diuresis was attempted. As a result, her creatinine increased initially but improved on repeated evaluation, and the patient had a new baseline creatinine of 1.6. Overall she had a net change in the fluid status of 10 liters negative by her ten days of admission in the ICU.

Mildly anemic otherwise, WBC and platelet counts were normal. Electrolyte balance should be monitored closely, paying attention to sodium, potassium, chloride, and calcium specifically as these are worsened in both renal failure and myxedema. 

Daily sedation vacations were enacted, and the patient's mental status improved and was much better when TSH was around 20. The bilateral pleural effusions improved with aggressive diuresis. Breathing trials were initiated when the patient's fio2 requirements decreased to 60% and a PEEP of 8. She was eventually extubated onto BiPAP and then high-flow nasal cannula while off of BiPAP. Pericardial fluid remained stable, and no cardiac tamponade pathology developed. As a result, it was determined that a pericardial window was unnecessary. Furthermore, she was not a candidate for pericardiocentesis as the pericardial effusion was located posterior to the heart. Her renal failure improved with improved cardiac function, diuretics, and thyroid hormone replacement.

After extubation patient had speech and swallow evaluations and was able to resume an oral diet. The patient was eventually transferred out of the ICU to the general medical floor and eventually to a rehabilitation unit.

Despite the name myxedema coma, most patients will not present in a coma status. This illness is at its core a severe hypothyroidism crisis that leads to systemic multiorgan failure. Thyroid hormones T3, and to a lesser extent, T4 act directly on a cellular level to upregulate all metabolic processes in the body. Therefore, deficiency of this hormone is characterized by systemic decreased metabolism and decreased glucose utilization along with increased production and storage of osmotically active mucopolysaccharide protein complexes into peripheral tissues resulting in diffuse edema and swelling of tissue. [1]

Myxedema coma is an illness that occurs primarily in females at a rate of 4:1 compared to men. It typically impacts the elderly at the age of greater than 60 years old, and approximately 90% of cases occur during the winter months. Myxedema coma is the product of longstanding unidentified or undertreated hypothyroidism of any etiology. Thyroid hormone is necessary throughout the body and acts as a regulatory hormone that affects many organ systems. [2] In cardiac tissues, myxedema coma manifests as decreased contractility with subsequent reduction in stroke volume and overall cardiac output.  Bradycardia and hypotension are typically present also. Pericardial effusions occur due to the accumulation of mucopolysaccharides in the pericardial sac, which leads to worsened cardiac function and congestive heart failure from diastolic dysfunction. Capillary permeability is also increased throughout the body leading to worsened edema. Electrocardiogram findings may include bradycardia and low-voltage, non-specific ST waveform changes with possible inverted T waves.

Neurologic tissues are impacted in myxedema coma leading to the pathognomonic altered mental status resulting from hypoxia and decreased cerebral blood flow secondary to cardiac dysfunction as above. Additionally, hypothyroidism leads to decreased glucose uptake and utilization in neurological tissue, thus worsening cognitive function.

The pulmonary system typically manifests this disease process through hypoventilation secondary to the central nervous system (CNS) depression of the respiratory drive with blunting of the response to hypoxia and hypercapnia. Additionally, metabolic dysfunction in the muscles of respiration leads to respiratory fatigue and failure, macroglossia from mucopolysaccharide driven edema of the tongue leads to mechanical obstruction of the airway, and obesity hypoventilation syndrome with the decreased respiratory drive as most hypothyroid patients suffer from obesity.

Renal manifestations include decreased glomerular filtration rate from the reduced cardiac output and increased systemic vascular resistance coupled with acute rhabdomyolysis lead to acute kidney injury. In the case of our patient above who has a pre-existing renal disease status post-nephrectomy, this is further worsened.  The net effect is worsened fluid overload status compounding the cardiac dysfunction and edema. [3]

The gastrointestinal tract is marked by mucopolysaccharide-driven edema as well leading to malabsorption of nutrients, gastric ileus, and decreased peristalsis. Ascites is common because of increased capillary permeability in the intestines coupled with coexistent congestive heart failure and congestive hepatic failure. Coagulopathies are common to occur as a result of this hepatic dysfunction.

Evaluation: The diagnosis of myxedema coma, as with all other diseases, is heavily reliant on the history and physical exam. A past medical history including hypothyroidism is highly significant whenever decreased mental status or coma is identified. In the absence of identified hypothyroidism, myxedema coma is a diagnosis of exclusion when all other sources of coma have been ruled out. If myxedema coma is suspected, evaluation of thyroid-stimulating hormone (TSH), free thyroxine (T4), and serum cortisol is warranted. T4 will be extremely low. TSH is variable depending on the etiology of hypothyroidism, with a high TSH indicating primary hypothyroidism and a low or normal TSH indicating secondary etiologies. Cortisol may be low indicating adrenal insufficiency because of hypothyroidism.  [4]

Prognosis: Myxedema coma is a medical emergency. With proper and rapid diagnosis and initiation of therapy, the mortality rate is still as high as 25% to 50%. The most common cause of death is due to respiratory failure. The factors which suggest a poorer prognosis include increased age, persistent hypothermia, bradycardia, low score Glasgow Coma Scale, or multi-organ impairment indicated by high APACHE (Acute Physiology and Chronic Health Evaluation) II score. For these reasons, placement in an intensive care unit with a low threshold for intubation and mechanical ventilation can improve mortality outcomes. [3] [5]

  • Pearls of Wisdom
  • Not every case of shortness of breath is COPD or congestive heart failure (CHF). While less likely, a history of hypothyroidism should raise suspicion of myxedema coma in a patient with any cognitive changes.
  • Myxedema is the great imitator illness that impacts all organ systems. It can easily be mistaken for congestive heart failure, COPD exacerbation, pneumonia, renal injury or failure, or neurological insult.
  • Initial steps in therapy include aggressive airway management, thyroid hormone replacement, glucocorticoid therapy, and supportive measures.
  • These patients should be monitored in an intensive care environment with continuous telemetry. [6]
  • Enhancing Healthcare Team Outcomes

This case demonstrates how all interprofessional healthcare team members need to be involved in arriving at a correct diagnosis, particularly in more challenging cases such as this one. Clinicians, specialists, nurses, pharmacists, laboratory technicians all bear responsibility for carrying out the duties pertaining to their particular discipline and sharing any findings with all team members. An incorrect diagnosis will almost inevitably lead to incorrect treatment, so coordinated activity, open communication, and empowerment to voice concerns are all part of the dynamic that needs to drive such cases so patients will attain the best possible outcomes.

  • Review Questions
  • Access free multiple choice questions on this topic.
  • Comment on this article.

Case Study of 60 year old female presenting with Shortness of Breath Contributed by Sandeep Sharma, MD

Disclosure: Deepa Rawat declares no relevant financial relationships with ineligible companies.

Disclosure: Sandeep Sharma declares no relevant financial relationships with ineligible companies.

This book is distributed under the terms of the Creative Commons Attribution-NonCommercial-NoDerivatives 4.0 International (CC BY-NC-ND 4.0) ( http://creativecommons.org/licenses/by-nc-nd/4.0/ ), which permits others to distribute the work, provided that the article is not altered or used commercially. You are not required to obtain permission to distribute this article, provided that you credit the author and journal.

  • Cite this Page Rawat D, Sharma S. Case Study: 60-Year-Old Female Presenting With Shortness of Breath. [Updated 2023 Feb 20]. In: StatPearls [Internet]. Treasure Island (FL): StatPearls Publishing; 2024 Jan-.

In this Page

Bulk download.

  • Bulk download StatPearls data from FTP

Related information

  • PubMed Links to PubMed

Similar articles in PubMed

  • Case Study: 33-Year-Old Female Presents with Chronic SOB and Cough. [StatPearls. 2024] Case Study: 33-Year-Old Female Presents with Chronic SOB and Cough. Sharma S, Hashmi MF, Rawat D. StatPearls. 2024 Jan
  • Acute Exacerbation of COPD. [J Educ Teach Emerg Med. 2023] Acute Exacerbation of COPD. Pappas D, Vempati A. J Educ Teach Emerg Med. 2023 Apr; 8(2):S35-S61. Epub 2023 Apr 30.
  • Review Breathlessness with pulmonary metastases: a multimodal approach. [J Adv Pract Oncol. 2013] Review Breathlessness with pulmonary metastases: a multimodal approach. Brant JM. J Adv Pract Oncol. 2013 Nov; 4(6):415-22.
  • Stress Cardiomyopathy in the Setting of COPD Exacerbation. [J Investig Med High Impact Cas...] Stress Cardiomyopathy in the Setting of COPD Exacerbation. Landefeld K, Saleh Q, Sander GE. J Investig Med High Impact Case Rep. 2015 Oct-Dec; 3(4):2324709615612847. Epub 2015 Oct 14.
  • Review Palliative Chemotherapy: Does It Only Provide False Hope? The Role of Palliative Care in a Young Patient With Newly Diagnosed Metastatic Adenocarcinoma. [J Adv Pract Oncol. 2017] Review Palliative Chemotherapy: Does It Only Provide False Hope? The Role of Palliative Care in a Young Patient With Newly Diagnosed Metastatic Adenocarcinoma. Doverspike L, Kurtz S, Selvaggi K. J Adv Pract Oncol. 2017 May-Jun; 8(4):382-386. Epub 2017 May 1.

Recent Activity

  • Case Study: 60-Year-Old Female Presenting With Shortness of Breath - StatPearls Case Study: 60-Year-Old Female Presenting With Shortness of Breath - StatPearls

Your browsing activity is empty.

Activity recording is turned off.

Turn recording back on

Connect with NLM

National Library of Medicine 8600 Rockville Pike Bethesda, MD 20894

Web Policies FOIA HHS Vulnerability Disclosure

Help Accessibility Careers

statistics

Case Study: Managing Severe Asthma in an Adult

—he follows his treatment plan, but this 40-year-old male athlete has asthma that is not well-controlled. what’s the next step.

By Kirstin Bass, MD, PhD Reviewed by Michael E. Wechsler, MD, MMSc

This case presents a patient with poorly controlled asthma that remains refractory to treatment despite use of standard-of-care therapeutic options. For patients such as this, one needs to embark on an extensive work-up to confirm the diagnosis, assess for comorbidities, and finally, to consider different therapeutic options.

image

Case presentation and patient history

Mr. T is a 40-year-old recreational athlete with a medical history significant for asthma, for which he has been using an albuterol rescue inhaler approximately 3 times per week for the past year. During this time, he has also been waking up with asthma symptoms approximately twice a month, and has had three unscheduled asthma visits for mild flares. Based on the  National Asthma Education and Prevention Program guidelines , Mr. T has asthma that is not well controlled. 1

As a result of these symptoms, spirometry was performed revealing a forced expiratory volume in the first second (FEV1) of 78% predicted. Mr. T then was prescribed treatment with a low-dose corticosteroid, fluticasone 44 mcg at two puffs twice per day. However, he remained symptomatic and continued to use his rescue inhaler 3 times per week. Therefore, he was switched to a combination inhaled steroid and long-acting beta-agonist (LABA) (fluticasone propionate 250 mcg and salmeterol 50 mcg, one puff twice a day) by his primary care doctor.

Initial pulmonary assessment Even with this step up in his medication, Mr. T continued to be symptomatic and require rescue inhaler use. Therefore, he was referred to a pulmonologist, who performed the initial work-up shown here:

  • Spirometry, pre-albuterol: FEV1 79%, post-albuterol: 12% improvement
  • Methacholine challenge: PC 20 : 1.0 mg/mL
  • Chest X-ray: Within normal limits

Continued pulmonary assessment His dose of inhaled corticosteroid (ICS) and LABA was increased to fluticasone 500 mcg/salmeterol 50 mcg, one puff twice daily. However, he continued to have symptoms and returned to the pulmonologist for further work-up, shown here:

  • Chest computed tomography (CT): Normal lung parenchyma with no scarring or bronchiectasis
  • Sinus CT: Mild mucosal thickening
  • Complete blood count (CBC): Within normal limits, white blood cells (WBC) 10.0 K/mcL, 3% eosinophils
  • Immunoglobulin E (IgE): 25 IU/mL
  • Allergy-skin test: Positive for dust, trees
  • Exhaled NO: Fractional exhaled nitric oxide (FeNO) 53 parts per billion (pbb)

Assessment for comorbidities contributing to asthma symptoms After this work-up, tiotropium was added to his medication regimen. However, he remained symptomatic and had two more flares over the next 3 months. He was assessed for comorbid conditions that might be affecting his symptoms, and results showed:

  • Esophagram/barium swallow: Negative
  • Esophageal manometry: Negative
  • Esophageal impedance: Within normal limits
  • ECG: Within normal limits
  • Genetic testing: Negative for cystic fibrosis, alpha1 anti-trypsin deficiency

The ear, nose, and throat specialist to whom he was referred recommended only nasal inhaled steroids for his mild sinus disease and noted that he had a normal vocal cord evaluation.

Following this extensive work-up that transpired over the course of a year, Mr. T continued to have symptoms. He returned to the pulmonologist to discuss further treatment options for his refractory asthma.

Diagnosis Mr. T has refractory asthma. Work-up for this condition should include consideration of other causes for the symptoms, including allergies, gastroesophageal reflux disease, cardiac disease, sinus disease, vocal cord dysfunction, or genetic diseases, such as cystic fibrosis or alpha1 antitrypsin deficiency, as was performed for Mr. T by his pulmonary team.

Treatment options When a patient has refractory asthma, treatment options to consider include anticholinergics (tiotropium, aclidinium), leukotriene modifiers (montelukast, zafirlukast), theophylline, anti-immunoglobulin E (IgE) antibody therapy with omalizumab, antibiotics, bronchial thermoplasty, or enrollment in a clinical trial evaluating the use of agents that modulate the cell signaling and immunologic responses seen in asthma.

Treatment outcome Mr. T underwent bronchial thermoplasty for his asthma. One year after the procedure, he reports feeling great. He has not taken systemic steroids for the past year, and his asthma remains controlled on a moderate dose of ICS and a LABA. He has also been able to resume exercising on a regular basis.

Approximately 10% to 15% of asthma patients have severe asthma refractory to the commonly available medications. 2  One key aspect of care for this patient population is a careful workup to exclude other comorbidities that could be contributing to their symptoms. Following this, there are several treatment options to consider, as in recent years there have been several advances in the development of asthma therapeutics. 2

Treatment options for refractory asthma There are a number of currently approved therapies for severe, refractory asthma. In addition to therapy with ICS or combination therapies with ICS and LABAs, leukotriene antagonists have good efficacy in asthma, especially in patients with prominent allergic or exercise symptoms. 2  The anticholinergics, such as tiotropium, which was approved for asthma in 2015, enhance bronchodilation and are useful adjuncts to ICS. 3-5  Omalizumab is a monoclonal antibody against IgE recommended for use in severe treatment-refractory allergic asthma in patients with atopy. 2  A nonmedication therapeutic option to consider is bronchial thermoplasty, a bronchoscopic procedure that uses thermal energy to disrupt bronchial smooth muscle. 6,7

Personalizing treatment for each patient It is important to personalize treatment based on individual characteristics or phenotypes that predict the patient's likely response to treatment, as well as the patient's preferences and practical issues, such as adherence and cost. 8

In this case, tiotropium had already been added to Mr. T's medications and his symptoms continued. Although addition of a leukotriene modifier was an option for him, he did not wish to add another medication to his care regimen. Omalizumab was not added partly for this reason, and also because of his low IgE level. As his bronchoscopy was negative, it was determined that a course of antibiotics would not be an effective treatment option for this patient. While vitamin D insufficiency has been associated with adverse outcomes in asthma, T's vitamin D level was tested and found to be sufficient.

We discussed the possibility of Mr. T's enrollment in a clinical trial. However, because this did not guarantee placement within a treatment arm and thus there was the possibility of receiving placebo, he opted to undergo bronchial thermoplasty.

Bronchial thermoplasty  Bronchial thermoplasty is effective for many patients with severe persistent asthma, such as Mr. T. This procedure may provide additional benefits to, but does not replace, standard asthma medications. During the procedure, thermal energy is delivered to the airways via a bronchoscope to reduce excess airway smooth muscle and limit its ability to constrict the airways. It is an outpatient procedure performed over three sessions by a trained physician. 9

The effects of bronchial thermoplasty have been studied in several trials. The first large-scale multicenter randomized controlled study was  the Asthma Intervention Research (AIR) Trial , which enrolled patients with moderate to severe asthma. 10  In this trial, patients who underwent the procedure had a significant improvement in asthma symptoms as measured by symptom-free days and scores on asthma control and quality of life questionnaires, as well as reductions in mild exacerbations and increases in morning peak expiratory flow. 10  Shortly after the AIR trial, the  Research in Severe Asthma (RISA) trial  was conducted to evaluate bronchial thermoplasty in patients with more severe, symptomatic asthma. 11  In this population, bronchial thermoplasty resulted in a transient worsening of asthma symptoms, with a higher rate of hospitalizations during the treatment period. 11  Hospitalization rate equalized between the treatment and control groups in the posttreatment period, however, and the treatment group showed significant improvements in rescue medication use, prebronchodilator forced expiratory volume in the first second (FEV1) % predicted, and asthma control questionnaire scores. 11

The AIR-2  trial followed, which was a multicenter, randomized, double-blind, sham-controlled study of 288 patients with severe asthma. 6  Similar to the RISA trial, patients in the treatment arm of this trial experienced an increase in adverse respiratory effects during the treatment period, the most common being airway irritation (including wheezing, chest discomfort, cough, and chest pain) and upper respiratory tract infections. 6

The majority of adverse effects occurred within 1 day of the procedure and resolved within 7 days. 6  In this study, bronchial thermoplasty was found to significantly improve quality of life, as well as reduce the rate of severe exacerbations by 32%. 6  Patients who underwent the procedure also reported fewer adverse respiratory effects, fewer days lost from work, school, or other activities due to asthma, and an 84% risk reduction in emergency department visits. 6

Long-term (5-year) follow-up studies have been conducted for patients in both  the AIR  and  the AIR-2  trials. In patients who underwent bronchial thermoplasty in either study, the rate of adverse respiratory effects remained stable in years 2 to 5 following the procedure, with no increase in hospitalizations or emergency department visits. 7,12  Additionally, FEV1 remained stable throughout the 5-year follow-up period. 7,12  This finding was maintained in patients enrolled in the AIR-2 trial despite decreased use of daily ICS. 7

Bronchial thermoplasty is an important addition to the asthma treatment armamentarium. 7  This treatment is currently approved for individuals with severe persistent asthma who remain uncontrolled despite the use of an ICS and LABA. Several clinical trials with long-term follow-up have now demonstrated its safety and ability to improve quality of life in patients with severe asthma, such as Mr. T.

Severe asthma can be a challenge to manage. Patients with this condition require an extensive workup, but there are several treatments currently available to help manage these patients, and new treatments are continuing to emerge. Managing severe asthma thus requires knowledge of the options available as well as consideration of a patient's personal situation-both in terms of disease phenotype and individual preference. In this case, the patient expressed a strong desire to not add any additional medications to his asthma regimen, which explained the rationale for choosing to treat with bronchial thermoplasty. Personalized treatment necessitates exploring which of the available or emerging options is best for each individual patient.

Published: April 16, 2018

  • 1. National Asthma Education and Prevention Program: Asthma Care Quick Reference.
  • 2. Olin JT, Wechsler ME. Asthma: pathogenesis and novel drugs for treatment. BMJ . 2014;349:g5517.
  • 3. Boehringer Ingelheim. Asthma: U.S. FDA approves new indication for SPIRIVA Respimat [press release]. September 16, 2015.
  • 4. Peters SP, Kunselman SJ, Icitovic N, et al. Tiotropium bromide step-up therapy for adults with uncontrolled asthma. N Engl J Med . 2010;363:1715-1726.
  • 5. Kerstjens HA, Engel M, Dahl R. Tiotropium in asthma poorly controlled with standard combination therapy. N Engl J Med . 2012;367:1198-1207.
  • 6. Castro M, Rubin AS, Laviolette M, et al. Effectiveness and safety of bronchial thermoplasty in the treatment of severe asthma: a multicenter, randomized, double-blind, sham-controlled clinical trial. Am J Respir Crit Care Med . 2010;181:116-124.
  • 7. Wechsler ME, Laviolette M, Rubin AS, et al. Bronchial thermoplasty: long-term safety and effectiveness in patients with severe persistent asthma. J Allergy Clin Immunol . 2013;132:1295-1302.
  • 8. Global Initiative for Asthma: Pocket Guide for Asthma Management and Prevention (for Adults and Children Older than 5 Years).
  • 10. Cox G, Thomson NC, Rubin AS, et al. Asthma control during the year after bronchial thermoplasty. N Engl J Med . 2007;356:1327-1337.
  • 11. Pavord ID, Cox G, Thomson NC, et al. Safety and efficacy of bronchial thermoplasty in symptomatic, severe asthma. Am J Respir Crit Care Med . 2007;176:1185-1191.
  • 12. Thomson NC, Rubin AS, Niven RM, et al. Long-term (5 year) safety of bronchial thermoplasty: Asthma Intervention Research (AIR) trial. BMC Pulm Med . 2011;11:8.

More On This Topic

Treatable traits and future exacerbation risk in severe asthma, baker’s asthma, the long-term trajectory of mild asthma, age, gender, & systemic corticosteroid comorbidities, ask the expert: william busse, md, challenges the current definition of the atopic march, considering the curveballs in asthma treatment, do mucus plugs play a bigger role in chronic severe asthma than previously thought, an emerging subtype of copd is associated with early respiratory disease.

image

Thank you for visiting nature.com. You are using a browser version with limited support for CSS. To obtain the best experience, we recommend you use a more up to date browser (or turn off compatibility mode in Internet Explorer). In the meantime, to ensure continued support, we are displaying the site without styles and JavaScript.

  • View all journals
  • My Account Login
  • Explore content
  • About the journal
  • Publish with us
  • Sign up for alerts
  • Perspective
  • Open access
  • Published: 16 October 2014

A woman with asthma: a whole systems approach to supporting self-management

  • Hilary Pinnock 1 ,
  • Elisabeth Ehrlich 1 ,
  • Gaylor Hoskins 2 &
  • Ron Tomlins 3  

npj Primary Care Respiratory Medicine volume  24 , Article number:  14063 ( 2014 ) Cite this article

16k Accesses

2 Citations

6 Altmetric

Metrics details

  • Health care

A 35-year-old lady attends for review of her asthma following an acute exacerbation. There is an extensive evidence base for supported self-management for people living with asthma, and international and national guidelines emphasise the importance of providing a written asthma action plan. Effective implementation of this recommendation for the lady in this case study is considered from the perspective of a patient, healthcare professional, and the organisation. The patient emphasises the importance of developing a partnership based on honesty and trust, the need for adherence to monitoring and regular treatment, and involvement of family support. The professional considers the provision of asthma self-management in the context of a structured review, with a focus on a self-management discussion which elicits the patient’s goals and preferences. The organisation has a crucial role in promoting, enabling and providing resources to support professionals to provide self-management. The patient’s asthma control was assessed and management optimised in two structured reviews. Her goal was to avoid disruption to her work and her personalised action plan focused on achieving that goal.

Similar content being viewed by others

nursing case study examples asthma

Barriers to implementing asthma self-management in Malaysian primary care: qualitative study exploring the perspectives of healthcare professionals

Ping Yein Lee, Ai Theng Cheong, … Ee Ming Khoo

nursing case study examples asthma

The self-management abilities test (SMAT): a tool to identify the self-management abilities of adults with bronchiectasis

Katelyn R. Smalley, Lisa Aufegger, … Ara Darzi

nursing case study examples asthma

Improving primary care management of asthma: do we know what really works?

Monica J. Fletcher, Ioanna Tsiligianni, … Thys van der Molen

A 35-year-old sales representative attends the practice for an asthma review. Her medical record notes that she has had asthma since childhood, and although for many months of the year her asthma is well controlled (when she often reduces or stops her inhaled steroids), she experiences one or two exacerbations a year requiring oral steroids. These are usually triggered by a viral upper respiratory infection, though last summer when the pollen count was particularly high she became tight chested and wheezy for a couple of weeks.

Her regular prescription is for fluticasone 100 mcg twice a day, and salbutamol as required. She has a young family and a busy lifestyle so does not often manage to find time to attend the asthma clinic. A few weeks previously, an asthma attack had interfered with some important work-related travel, and she has attended the clinic on this occasion to ask about how this can be managed better in the future. There is no record of her having been given an asthma action plan.

What do we know about asthma self-management? The academic perspective

Supported self-management reduces asthma morbidity.

The lady in this case study is struggling to maintain control of her asthma within the context of her busy professional and domestic life. The recent unfortunate experience which triggered this consultation offers a rare opportunity to engage with her and discuss how she can manage her asthma better. It behoves the clinician whom she is seeing (regardless of whether this is in a dedicated asthma clinic or an appointment in a routine general practice surgery) to grasp the opportunity and discuss self-management and provide her with a (written) personalised asthma action plan (PAAP).

The healthcare professional advising the lady is likely to be aware that international and national guidelines emphasise the importance of supporting self-management. 1 – 4 There is an extensive evidence base for asthma self-management: a recent synthesis identified 22 systematic reviews summarising data from 260 randomised controlled trials encompassing a broad range of demographic, clinical and healthcare contexts, which concluded that asthma self-management reduces emergency use of healthcare resources, including emergency department visits, hospital admissions and unscheduled consultations and improves markers of asthma control, including reduced symptoms and days off work, and improves quality of life. 1 , 2 , 5 – 12 Health economic analysis suggests that it is not only clinically effective, but also a cost-effective intervention. 13

Personalised asthma action plans

Key features of effective self-management approaches are:

Self-management education should be reinforced by provision of a (written) PAAP which reminds patients of their regular treatment, how to monitor and recognise that control is deteriorating and the action they should take. 14 – 16 As an adult, our patient can choose whether she wishes to monitor her control with symptoms or by recording peak flows (or a combination of both). 6 , 8 , 9 , 14 Symptom-based monitoring is generally better in children. 15 , 16

Plans should have between two and three action points including emergency doses of reliever medication; increasing low dose (or recommencing) inhaled steroids; or starting a course of oral steroids according to severity of the exacerbation. 14

Personalisation of the action plan is crucial. Focussing specifically on what actions she could take to prevent a repetition of the recent attack is likely to engage her interest. Not all patients will wish to start oral steroids without advice from a healthcare professional, though with her busy lifestyle and travel our patient is likely to be keen to have an emergency supply of prednisolone. Mobile technology has the potential to support self-management, 17 , 18 though a recent systematic review concluded that none of the currently available smart phone ‘apps’ were fit for purpose. 19

Identification and avoidance of her triggers is important. As pollen seems to be a trigger, management of allergic rhinitis needs to be discussed (and included in her action plan): she may benefit from regular use of a nasal steroid spray during the season. 20

Self-management as recommended by guidelines, 1 , 2 focuses narrowly on adherence to medication/monitoring and the early recognition/remediation of exacerbations, summarised in (written) PAAPs. Patients, however, may want to discuss how to reduce the impact of asthma on their life more generally, 21 including non-pharmacological approaches.

Supported self-management

The impact is greater if self-management education is delivered within a comprehensive programme of accessible, proactive asthma care, 22 and needs to be supported by ongoing regular review. 6 With her busy lifestyle, our patient may be reluctant to attend follow-up appointments, and once her asthma is controlled it may be possible to make convenient arrangements for professional review perhaps by telephone, 23 , 24 or e-mail. Flexible access to professional advice (e.g., utilising diverse modes of consultation) is an important component of supporting self-management. 25

The challenge of implementation

Implementation of self-management, however, remains poor in routine clinical practice. A recent Asthma UK web-survey estimated that only 24% of people with asthma in the UK currently have a PAAP, 26 with similar figures from Sweden 27 and Australia. 28 The general practitioner may feel that they do not have time to discuss self-management in a routine surgery appointment, or may not have a supply of paper-based PAAPs readily available. 29 However, as our patient rarely finds time to attend the practice, inviting her to make an appointment for a future clinic is likely to be unsuccessful and the opportunity to provide the help she needs will be missed.

The solution will need a whole systems approach

A systematic meta-review of implementing supported self-management in long-term conditions (including asthma) concluded that effective implementation was multifaceted and multidisciplinary; engaging patients, training and motivating professionals within the context of an organisation which actively supported self-management. 5 This whole systems approach considers that although patient education, professional training and organisational support are all essential components of successful support, they are rarely effective in isolation. 30 A systematic review of interventions that promote provision/use of PAAPs highlighted the importance of organisational systems (e.g., sending blank PAAPs with recall reminders). 31 A patient offers her perspective ( Box 1 ), a healthcare professional considers the clinical challenge, and the challenges are discussed from an organisational perspective.

Box 1: What self-management help should this lady expect from her general practitioner or asthma nurse? The patient’s perspective

The first priority is that the patient is reassured that her condition can be managed successfully both in the short and the long term. A good working relationship with the health professional is essential to achieve this outcome. Developing trust between patient and healthcare professional is more likely to lead to the patient following the PAAP on a long-term basis.

A review of all medication and possible alternative treatments should be discussed. The patient needs to understand why any changes are being made and when she can expect to see improvements in her condition. Be honest, as sometimes it will be necessary to adjust dosages before benefits are experienced. Be positive. ‘There are a number of things we can do to try to reduce the impact of asthma on your daily life’. ‘Preventer treatment can protect against the effect of pollen in the hay fever season’. If possible, the same healthcare professional should see the patient at all follow-up appointments as this builds trust and a feeling of working together to achieve the aim of better self-management.

Is the healthcare professional sure that the patient knows how to take her medication and that it is taken at the same time each day? The patient needs to understand the benefit of such a routine. Medication taken regularly at the same time each day is part of any self-management regime. If the patient is unused to taking medication at the same time each day then keeping a record on paper or with an electronic device could help. Possibly the patient could be encouraged to set up a system of reminders by text or smartphone.

Some people find having a peak flow meter useful. Knowing one's usual reading means that any fall can act as an early warning to put the PAAP into action. Patients need to be proactive here and take responsibility.

Ongoing support is essential for this patient to ensure that she takes her medication appropriately. Someone needs to be available to answer questions and provide encouragement. This could be a doctor or a nurse or a pharmacist. Again, this is an example of the partnership needed to achieve good asthma control.

It would also be useful at a future appointment to discuss the patient’s lifestyle and work with her to reduce her stress. Feeling better would allow her to take simple steps such as taking exercise. It would also be helpful if all members of her family understood how to help her. Even young children can do this.

From personal experience some people know how beneficial it is to feel they are in a partnership with their local practice and pharmacy. Being proactive produces dividends in asthma control.

What are the clinical challenges for the healthcare professional in providing self-management support?

Due to the variable nature of asthma, a long-standing history may mean that the frequency and severity of symptoms, as well as what triggers them, may have changed over time. 32 Exacerbations requiring oral steroids, interrupting periods of ‘stability’, indicate the need for re-assessment of the patient’s clinical as well as educational needs. The patient’s perception of stability may be at odds with the clinical definition 1 , 33 —a check on the number of short-acting bronchodilator inhalers the patient has used over a specific period of time is a good indication of control. 34 Assessment of asthma control should be carried out using objective tools such as the Asthma Control Test or the Royal College of Physicians three questions. 35 , 36 However, it is important to remember that these assessment tools are not an end in themselves but should be a springboard for further discussion on the nature and pattern of symptoms. Balancing work with family can often make it difficult to find the time to attend a review of asthma particularly when the patient feels well. The practice should consider utilising other means of communication to maintain contact with patients, encouraging them to come in when a problem is highlighted. 37 , 38 Asthma guidelines advocate a structured approach to ensure the patient is reviewed regularly and recommend a detailed assessment to enable development of an appropriate patient-centred (self)management strategy. 1 – 4

Although self-management plans have been shown to be successful for reducing the impact of asthma, 21 , 39 the complexity of managing such a fluctuating disease on a day-to-day basis is challenging. During an asthma review, there is an opportunity to work with the patient to try to identify what triggers their symptoms and any actions that may help improve or maintain control. 38 An integral part of personalised self-management education is the written PAAP, which gives the patient the knowledge to respond to the changes in symptoms and ensures they maintain control of their asthma within predetermined parameters. 9 , 40 The PAAP should include details on how to monitor asthma, recognise symptoms, how to alter medication and what to do if the symptoms do not improve. The plan should include details on the treatment to be taken when asthma is well controlled, and how to adjust it when the symptoms are mild, moderate or severe. These action plans need to be developed between the doctor, nurse or asthma educator and the patient during the review and should be frequently reviewed and updated in partnership (see Box 1). Patient preference as well as clinical features such as whether she under- or over-perceives her symptoms should be taken into account when deciding whether the action plan is peak flow or symptom-driven. Our patient has a lot to gain from having an action plan. She has poorly controlled asthma and her lifestyle means that she will probably see different doctors (depending who is available) when she needs help. Being empowered to self-manage could make a big difference to her asthma control and the impact it has on her life.

The practice should have protocols in place, underpinned by specific training to support asthma self-management. As well as ensuring that healthcare professionals have appropriate skills, this should include training for reception staff so that they know what action to take if a patient telephones to say they are having an asthma attack.

However, focusing solely on symptom management strategies (actions) to follow in the presence of deteriorating symptoms fails to incorporate the patients’ wider views of asthma, its management within the context of her/his life, and their personal asthma management strategies. 41 This may result in a failure to use plans to maximise their health potential. 21 , 42 A self-management strategy leading to improved outcomes requires a high level of patient self-efficacy, 43 a meaningful partnership between the patient and the supporting health professional, 42 , 44 and a focused self-management discussion. 14

Central to both the effectiveness and personalisation of action plans, 43 , 45 in particular the likelihood that the plan will lead to changes in patients’ day-to-day self-management behaviours, 45 is the identification of goals. Goals are more likely to be achieved when they are specific, important to patients, collaboratively set and there is a belief that these can be achieved. Success depends on motivation 44 , 46 to engage in a specific behaviour to achieve a valued outcome (goal) and the ability to translate the behavioural intention into action. 47 Action and coping planning increases the likelihood that patient behaviour will actually change. 44 , 46 , 47 Our patient has a goal: she wants to avoid having her work disrupted by her asthma. Her personalised action plan needs to explicitly focus on achieving that goal.

As providers of self-management support, health professionals must work with patients to identify goals (valued outcomes) that are important to patients, that may be achievable and with which they can engage. The identification of specific, personalised goals and associated feasible behaviours is a prerequisite for the creation of asthma self-management plans. Divergent perceptions of asthma and how to manage it, and a mismatch between what patients want/need from these plans and what is provided by professionals are barriers to success. 41 , 42

What are the challenges for the healthcare organisation in providing self-management support?

A number of studies have demonstrated the challenges for primary care physicians in providing ongoing support for people with asthma. 31 , 48 , 49 In some countries, nurses and other allied health professionals have been trained as asthma educators and monitor people with stable asthma. These resources are not always available. In addition, some primary care services are delivered in constrained systems where only a few minutes are available to the practitioner in a consultation, or where only a limited range of asthma medicines are available or affordable. 50

There is recognition that the delivery of quality care depends on the competence of the doctor (and supporting health professionals), the relationship between the care providers and care recipients, and the quality of the environment in which care is delivered. 51 This includes societal expectations, health literacy and financial drivers.

In 2001, the Australian Government adopted a programme developed by the General Practitioner Asthma Group of the National Asthma Council Australia that provided a structured approach to the implementation of asthma management guidelines in a primary care setting. 52 Patients with moderate-to-severe asthma were eligible to participate. The 3+ visit plan required confirmation of asthma diagnosis, spirometry if appropriate, assessment of trigger factors, consideration of medication and patient self-management education including provision of a written PAAP. These elements, including regular medical review, were delivered over three visits. Evaluation demonstrated that the programme was beneficial but that it was difficult to complete the third visit in the programme. 53 – 55 Accordingly, the programme, renamed the Asthma Cycle of Care, was modified to incorporate two visits. 56 Financial incentives are provided to practices for each patient who receives this service each year.

Concurrently, other programmes were implemented which support practice-based care. Since 2002, the National Asthma Council has provided best-practice asthma and respiratory management education to health professionals, 57 and this programme will be continuing to 2017. The general practitioner and allied health professional trainers travel the country to provide asthma and COPD updates to groups of doctors, nurses and community pharmacists. A number of online modules are also provided. The PACE (Physician Asthma Care Education) programme developed by Noreen Clark has also been adapted to the Australian healthcare system. 58 In addition, a pharmacy-based intervention has been trialled and implemented. 59

To support these programmes, the National Asthma Council ( www.nationalasthma.org.au ) has developed resources for use in practices. A strong emphasis has been on the availability of a range of PAAPs (including plans for using adjustable maintenance dosing with ICS/LABA combination inhalers), plans for indigenous Australians, paediatric plans and plans translated into nine languages. PAAPs embedded in practice computer systems are readily available in consultations, and there are easily accessible online paediatric PAAPs ( http://digitalmedia.sahealth.sa.gov.au/public/asthma/ ). A software package, developed in the UK, can be downloaded and used to generate a pictorial PAAP within the consultation. 60

One of the strongest drivers towards the provision of written asthma action plans in Australia has been the Asthma Friendly Schools programme. 61 , 62 Established with Australian Government funding and the co-operation of Education Departments of each state, the Asthma Friendly Schools programme engages schools to address and satisfy a set of criteria that establishes an asthma-friendly environment. As part of accreditation, the school requires that each child with asthma should have a written PAAP prepared by their doctor to assist (trained) staff in managing a child with asthma at school.

The case study continues...

The initial presentation some weeks ago was during an exacerbation of asthma, which may not be the best time to educate a patient. It is, however, a splendid time to build on their motivation to feel better. She agreed to return after her asthma had settled to look more closely at her asthma control, and an appointment was made for a routine review.

At this follow-up consultation, the patient’s diagnosis was reviewed and confirmed and her trigger factors discussed. For this lady, respiratory tract infections are the usual trigger but allergic factors during times of high pollen count may also be relevant. Assessment of her nasal airway suggested that she would benefit from better control of allergic rhinitis. Other factors were discussed, as many patients are unaware that changes in air temperature, exercise and pets can also trigger asthma exacerbations. In addition, use of the Asthma Control Test was useful as an objective assessment of control as well as helping her realise what her life could be like! Many people with long-term asthma live their life within the constraints of their illness, accepting that is all that they can do.

After assessing the level of asthma control, a discussion about management options—trigger avoidance, exercise and medicines—led to the development of a written PAAP. Asthma can affect the whole family, and ways were explored that could help her family understand why it is important that she finds time in the busy domestic schedules to take her regular medication. Family and friends can also help by understanding what triggers her asthma so that they can avoid exposing her to perfumes, pollens or pets that risk triggering her symptoms. Information from the national patient organisation was provided to reinforce the messages.

The patient agreed to return in a couple of weeks, and a recall reminder was set up. At the second consultation, the level of control since the last visit will be explored including repeat spirometry, if appropriate. Further education about the pathophysiology of asthma and how to recognise early warning signs of loss of control can be given. Device use will be reassessed and the PAAP reviewed. Our patient’s goal is to avoid disruption to her work and her PAAP will focus on achieving that goal. Finally, agreement will be reached with the patient about future routine reviews, which, now that she has a written PAAP, could be scheduled by telephone if all is well, or face-to-face if a change in her clinical condition necessitates a more comprehensive review.

Global Initiative for Asthma. Global Strategy for Asthma Management and Prevention, 2012. Available from: http://www.ginasthma.org (accessed July 2013).

British Thoracic Society/Scottish Intercollegiate Guideline Network British Guideline on the Management of Asthma. Thorax 2008; 63 (Suppl 4 iv1–121, updated version available from: http://www.sign.ac.uk (accessed January 2014).

Article   Google Scholar  

National Asthma Council Australia. Australian Asthma Handbook. Available from: http://www.nationalasthma.org.au/handbook (accessed May 2014).

National Asthma Education and Prevention Program (NAEPP) Coordinating Committee. Expert Panel Report 3 (EPR3): Guidelines for the Diagnosis and Management of Asthma. Available from: https://www.nhlbi.nih.gov/guidelines/asthma/asthgdln.htm (accessed May 2014).

Taylor SJC, Pinnock H, Epiphaniou E, Pearce G, Parke H . A rapid synthesis of the evidence on interventions supporting self-management for people with long-term conditions. (PRISMS Practical Systematic Review of Self-Management Support for long-term conditions). Health Serv Deliv Res (in press).

Gibson PG, Powell H, Wilson A, Abramson MJ, Haywood P, Bauman A et al. Self-management education and regular practitioner review for adults with asthma. Cochrane Database Syst Rev 2002: (Issue 3) Art No. CD001117.

Tapp S, Lasserson TJ, Rowe BH . Education interventions for adults who attend the emergency room for acute asthma. Cochrane Database Syst Rev 2007: (Issue 3) Art No. CD003000.

Powell H, Gibson PG . Options for self-management education for adults with asthma. Cochrane Database Syst Rev 2002: (Issue 3) Art No: CD004107.

Toelle B, Ram FSF . Written individualised management plans for asthma in children and adults. Cochrane Database Syst Rev 2004: (Issue 1) Art No. CD002171.

Lefevre F, Piper M, Weiss K, Mark D, Clark N, Aronson N . Do written action plans improve patient outcomes in asthma? An evidence-based analysis. J Fam Pract 2002; 51 : 842–848.

PubMed   Google Scholar  

Boyd M, Lasserson TJ, McKean MC, Gibson PG, Ducharme FM, Haby M . Interventions for educating children who are at risk of asthma-related emergency department attendance. Cochrane Database Syst Rev 2009: (Issue 2) Art No.CD001290.

Bravata DM, Gienger AL, Holty JE, Sundaram V, Khazeni N, Wise PH et al. Quality improvement strategies for children with asthma: a systematic review. Arch Pediatr Adolesc Med 2009; 163 : 572–581.

Bower P, Murray E, Kennedy A, Newman S, Richardson G, Rogers A . Self-management support interventions to reduce health care utilisation without compromising outcomes: a rapid synthesis of the evidence. Available from: http://www.nets.nihr.ac.uk/projects/hsdr/11101406 (accessed April 2014).

Gibson PG, Powell H . Written action plans for asthma: an evidence-based review of the key components. Thorax 2004; 59 : 94–99.

Article   CAS   Google Scholar  

Bhogal SK, Zemek RL, Ducharme F . Written action plans for asthma in children. Cochrane Database Syst Rev 2006: (Issue 3) Art No. CD005306.

Zemek RL, Bhogal SK, Ducharme FM . Systematic review of randomized controlled trials examining written action plans in children: what is the plan?. Arch Pediatr Adolesc Med 2008; 162 : 157–163.

Pinnock H, Slack R, Pagliari C, Price D, Sheikh A . Understanding the potential role of mobile phone based monitoring on asthma self-management: qualitative study. Clin Exp Allergy 2007; 37 : 794–802.

de Jongh T, Gurol-Urganci I, Vodopivec-Jamsek V, Car J, Atun R . Mobile phone messaging for facilitating self-management of long-term illnesses. Cochrane Database Syst Rev 2012: (Issue 12) Art No. CD007459.

Huckvale K, Car M, Morrison C, Car J . Apps for asthma self-management: a systematic assessment of content and tools. BMC Med 2012; 10 : 144.

Allergic Rhinitis and its Impact on Asthma. Management of Allergic Rhinitis and its Impact on Asthma: Pocket Guide. ARIA 2008. Available from: http://www.whiar.org (accessed May 2014).

Ring N, Jepson R, Hoskins G, Wilson C, Pinnock H, Sheikh A et al. Understanding what helps or hinders asthma action plan use: a systematic review and synthesis of the qualitative literature. Patient Educ Couns 2011; 85 : e131–e143.

Moullec G, Gour-Provencal G, Bacon SL, Campbell TS, Lavoie KL . Efficacy of interventions to improve adherence to inhaled corticosteroids in adult asthmatics: Impact of using components of the chronic care model. Respir Med 2012; 106 : 1211–1225.

Pinnock H, Bawden R, Proctor S, Wolfe S, Scullion J, Price D et al. Accessibility, acceptability and effectiveness of telephone reviews for asthma in primary care: randomised controlled trial. BMJ 2003; 326 : 477–479.

Pinnock H, Adlem L, Gaskin S, Harris J, Snellgrove C, Sheikh A . Accessibility, clinical effectiveness and practice costs of providing a telephone option for routine asthma reviews: phase IV controlled implementation study. Br J Gen Pract 2007; 57 : 714–722.

PubMed   PubMed Central   Google Scholar  

Kielmann T, Huby G, Powell A, Sheikh A, Price D, Williams S et al. From support to boundary: a qualitative study of the border between self care and professional care. Patient Educ Couns 2010; 79 : 55–61.

Asthma UK . Compare your care report. Asthma UK, 2013. Available from: http://www.asthma.org.uk (accessed January 2014).

Stallberg B, Lisspers K, Hasselgren M, Janson C, Johansson G, Svardsudd K . Asthma control in primary care in Sweden: a comparison between 2001 and 2005. Prim Care Respir J 2009; 18 : 279–286.

Reddel H, Peters M, Everett P, Flood P, Sawyer S . Ownership of written asthma action plans in a large Australian survey. Eur Respir J 2013; 42 . Abstract 2011.

Wiener-Ogilvie S, Pinnock H, Huby G, Sheikh A, Partridge MR, Gillies J . Do practices comply with key recommendations of the British Asthma Guideline? If not, why not? Prim Care Respir J 2007; 16 : 369–377.

Kennedy A, Rogers A, Bower P . Support for self care for patients with chronic disease. BMJ 2007; 335 : 968–970.

Ring N, Malcolm C, Wyke S, Macgillivray S, Dixon D, Hoskins G et al. Promoting the Use of Personal Asthma Action Plans: A Systematic Review. Prim Care Respir J 2007; 16 : 271–283.

Taylor DR, Bateman ED, Boulet LP, Boushey HA, Busse WW, Casale TB et al. A new perspective on concepts of asthma severity and control. Eur Respir J 2008; 32 : 545–554.

Horne R . Compliance, adherence, and concordance: implications for asthma treatment. Chest 2006; 130 (suppl): 65S–72S.

Reddel HK, Taylor DR, Bateman ED, Boulet L-P, Boushey HA, Busse WW et al. An official American Thoracic Society/European Respiratory Society statement: asthma control and exacerbations standardizing endpoints for clinical asthma trials and clinical practice. Am J Respir Crit Care Med 2009; 180 : 59–99.

Thomas M, Kay S, Pike J, Rosenzweig JR, Hillyer EV, Price D . The Asthma Control Test (ACT) as a predictor of GINA guideline-defined asthma control: analysis of a multinational cross-sectional survey. Prim Care Respir J 2009; 18 : 41–49.

Hoskins G, Williams B, Jackson C, Norman P, Donnan P . Assessing asthma control in UK primary care: use of routinely collected prospective observational consultation data to determine appropriateness of a variety of control assessment models. BMC Fam Pract 2011; 12 : 105.

Pinnock H, Fletcher M, Holmes S, Keeley D, Leyshon J, Price D et al. Setting the standard for routine asthma consultations: a discussion of the aims, process and outcomes of reviewing people with asthma in primary care. Prim Care Respir J 2010; 19 : 75–83.

McKinstry B, Hammersley V, Burton C, Pinnock H, Elton RA, Dowell J et al. The quality, safety and content of telephone and face-to-face consultations: a comparative study. Qual Saf Health Care 2010; 19 : 298–303.

Gordon C, Galloway T . Review of Findings on Chronic Disease Self-Management Program (CDSMP) Outcomes: Physical, Emotional & Health-Related Quality of Life, Healthcare Utilization and Costs . Centers for Disease Control and Prevention and National Council on Aging: Atlanta, GA, USA, 2008.

Beasley R, Crane J . Reducing asthma mortality with the self-management plan system of care. Am J Respir Crit Care Med 2001; 163 : 3–4.

Ring N, Jepson R, Pinnock H, Wilson C, Hoskins G, Sheikh A et al. Encouraging the promotion and use of asthma action plans: a cross study synthesis of qualitative and quantitative evidence. Trials 2012; 13 : 21.

Jones A, Pill R, Adams S . Qualitative study of views of health professionals and patients on guided self-management plans for asthma. BMJ 2000; 321 : 1507–1510.

Bandura A . Self-efficacy: toward a unifying theory of behavioural change. Psychol Rev 1977; 84 : 191–215.

Gollwitzer PM, Sheeran P . Implementation intentions and goal achievement: a meta-analysis of effects and processes. Adv Exp Soc Psychol 2006; 38 : 69–119.

Google Scholar  

Hardeman W, Johnston M, Johnston DW, Bonetti D, Wareham NJ, Kinmonth AL . Application of the theory of planned behaviour change interventions: a systematic review. Psychol Health 2002; 17 : 123–158.

Schwarzer R . Modeling health behavior change: how to predict and modify the adoption and maintenance of health behaviors. Appl Psychol 2008; 57 : 1–29.

Sniehotta F . Towards a theory of intentional behaviour change: plans, planning, and self-regulation. Br J Health Psychol 2009; 14 : 261–273.

Okelo SO, Butz AM, Sharma R, Diette GB, Pitts SI, King TM et al. Interventions to modify health care provider adherence to asthma guidelines: a systematic review. Pediatrics 2013; 132 : 517–534.

Grol R, Grimshaw RJ . From best evidence to best practice: effective implementation of change in patients care. Lancet 2003; 362 : 1225–1230.

Jusef L, Hsieh C-T, Abad L, Chaiyote W, Chin WS, Choi Y-J et al. Primary care challenges in treating paediatric asthma in the Asia-Pacific region. Prim Care Respir J 2013; 22 : 360–362.

Donabedian A . Evaluating the quality of medical care. Milbank Q 2005; 83 : 691–729.

Fardy HJ . Moving towards organized care of chronic disease. The 3+ visit plan. Aust Fam Physician 2001; 30 : 121–125.

CAS   PubMed   Google Scholar  

Glasgow NJ, Ponsonby AL, Yates R, Beilby J, Dugdale P . Proactive asthma care in childhood: general practice based randomised controlled trial. BMJ 2003; 327 : 659.

Douglass JA, Goemann DP, Abramson MJ . Asthma 3+ visit plan: a qualitative evaluation. Intern Med J 2005; 35 : 457–462.

Beilby J, Holton C . Chronic disease management in Australia; evidence and policy mismatch, with asthma as an example. Chronic Illn 2005; 1 : 73–80.

The Department of Health. Asthma Cycle of Care. Accessed on 14 May 2014 at http://www.health.gov.au/internet/main/publishing.nsf/Content/asthma-cycle .

National Asthma Council Australia. Asthma and Respiratory Education Program. Accessed on 14 May 2014 at http://www.nationalasthma.org.au/health-professionals/education-training/asthma-respiratory-education-program .

Patel MR, Shah S, Cabana MD, Sawyer SM, Toelle B, Mellis C et al. Translation of an evidence-based asthma intervention: Physician Asthma Care Education (PACE) in the United States and Australia. Prim Care Respir J 2013; 22 : 29–34.

Armour C, Bosnic-Anticevich S, Brilliant M, Burton D, Emmerton L, Krass I et al. Pharmacy Asthma Care Program (PACP) improves outcomes for patients in the community. Thorax 2007; 62 : 496–502.

Roberts NJ, Mohamed Z, Wong PS, Johnson M, Loh LC, Partridge MR . The development and comprehensibility of a pictorial asthma action plan. Patient Educ Couns 2009; 74 : 12–18.

Henry RL, Gibson PG, Vimpani GV, Francis JL, Hazell J . Randomised controlled trial of a teacher-led asthma education program. Pediatr Pulmonol 2004; 38 : 434–442.

National Asthma Council Australia. Asthma Friendly Schools program. Accessed on 14 May 2014 at http://www.asthmaaustralia.org.au/Asthma-Friendly-Schools.aspx .

Download references

Author information

Authors and affiliations.

Asthma UK Centre for Applied Research, Centre for Population Health Sciences, The University of Edinburgh, Edinburgh, UK,

Hilary Pinnock & Elisabeth Ehrlich

NMAHP-RU, University of Stirling, Stirling, UK,

Gaylor Hoskins

Discipline of General Practice, University of Sydney, Sydney, NSW, Australia

Ron Tomlins

You can also search for this author in PubMed   Google Scholar

Corresponding author

Correspondence to Hilary Pinnock .

Ethics declarations

Competing interests.

The authors declare no conflict of interest.

Rights and permissions

This work is licensed under a Creative Commons Attribution-NonCommercial-ShareAlike 4.0 International License. The images or other third party material in this article are included in the article's Creative Commons license, unless indicated otherwise in the credit line; if the material is not included under the Creative Commons license, users will need to obtain permission from the license holder to reproduce the material. To view a copy of this license, visit http://creativecommons.org/licenses/by-nc-sa/4.0/

Reprints and permissions

About this article

Cite this article.

Pinnock, H., Ehrlich, E., Hoskins, G. et al. A woman with asthma: a whole systems approach to supporting self-management. npj Prim Care Resp Med 24 , 14063 (2014). https://doi.org/10.1038/npjpcrm.2014.63

Download citation

Received : 23 June 2014

Revised : 15 July 2014

Accepted : 15 July 2014

Published : 16 October 2014

DOI : https://doi.org/10.1038/npjpcrm.2014.63

Share this article

Anyone you share the following link with will be able to read this content:

Sorry, a shareable link is not currently available for this article.

Provided by the Springer Nature SharedIt content-sharing initiative

Quick links

  • Explore articles by subject
  • Guide to authors
  • Editorial policies

Sign up for the Nature Briefing newsletter — what matters in science, free to your inbox daily.

nursing case study examples asthma

nursing case study examples asthma

Learn about the nursing care management of patients with asthma in this nursing study guide .

Table of Contents

  • What is Asthma? 

Pathophysiology

  • Statistics and Epidemiology

Clinical Manifestations

Complications, assessment and diagnostic findings, pharmacologic therapy, peak flow monitoring, nursing assessment, nursing diagnosis, nursing care planning & goals, nursing interventions, discharge and home care guidelines, documentation guidelines, practice quiz: asthma, what is asthma.

Asthma affects people in their different stages in life, yet it can be avoided and treated.

asthma-pathophysiology

  • Asthma is a chronic inflammatory disease of the airways that causes airway hyperresponsiveness, mucosal edema, and mucus production.
  • Inflammation ultimately leads to recurrent episodes of asthma symptoms.
  • Patients with asthma may experience symptom-free periods alternating with acute exacerbations that last from minutes to hours or days.
  • Asthma, the most common chronic disease of childhood, can begin at any age.

The underlying pathophysiology in asthma is reversible and diffuse airway inflammation that leads to airway narrowing.

  • Activation. When the mast cells are activated, it releases several chemicals called mediators.
  • Perpetuation. These chemicals perpetuate the inflammatory response, causing increased blood flow, vasoconstriction,, fluid leak from the vasculature, the attraction of white blood cells to the area, and bronchoconstriction.
  • Bronchoconstriction. Acute bronchoconstriction due to allergens results from a release of mediators from mast cells that directly contract the airway.
  • Progression. As asthma becomes more persistent, the inflammation progresses and other factors may be involved in the airflow limitation.

Statistics and Epidemiology

Asthma is considered the most common chronic disease of childhood and is a disruptive disease that affects school and work attendance.

  • Asthma affects more than 22 million people in the United States.
  • Asthma accounts for more than 497, 000 hospitalizations annually.
  • The total economic cost of asthma exceeds $27.6 billion.

Despite increased knowledge on the pathology of asthma and the development of improved medications and management plans, the death rate from the disease continues to rise. Here are some of the factors that influence the development of asthma.

  • Allergy . Allergy is the strongest predisposing factor for asthma.
  • Chronic exposure to airway irritants. Irritants can be seasonal (grass, tree, and weed pollens) or perennial (mold, dust, roaches, animal dander).
  • Exercise. Too much exercise can also cause asthma.
  • Stress/ Emotional upset. This can trigger constriction of the airway leading to asthma.
  • Medications. Certain medications can trigger asthma.

The signs and symptoms of asthma can be easily identified, so once the following symptoms are observed, a visit to the physician is necessary.

  • Most common symptoms of asthma are cough (with or without mucus production), dyspnea , and wheezing (first on expiration, then possibly during inspiration as well).
  • Cough . There are instances that cough is the only symptom.
  • Dyspnea. General tightness may occur which leads to dyspnea.
  • Wheezing. There may be wheezing, first on expiration, and then possibly during inspiration as well.
  • Asthma attacks frequently occur at night or in the early morning.
  • An asthma exacerbation is frequently preceded by increasing symptoms over days, but it may begin abruptly.
  • Expiration requires effort and becomes prolonged.
  • As exacerbation progresses, central cyanosis secondary to severe hypoxia may occur.
  • Additional symptoms, such as diaphoresis, tachycardia, and a widened pulse pressure, may occur.
  • Exercise-induced asthma: maximal symptoms during exercise, absence of nocturnal symptoms, and sometimes only a description of a “choking” sensation during exercise.
  • A severe, continuous reaction, status asthmaticus, may occur. It is life-threatening.
  • Eczema, rashes, and temporary edema are allergic reactions that may be noted with asthma.

Patients with recurrent asthma should undergo tests to identify the substances that precipitate the symptoms.

  • Allergens . Allergens, either seasonal or perennial, can be prevented through avoiding contact with them whenever possible.
  • Knowledge. Knowledge is the key to quality asthma care.
  • Evaluation. Evaluation of impairment and risk are key in the control.

Complications for asthma include the following:

  • Status asthmaticus . Airway obstruction in status asthmaticus often results in hypoxemia .
  • Respiratory failure . Asthma, if left untreated, progresses to respiratory failure.
  • Pneumonia . Mucus that pools in the lungs and becomes infected can lead to the development of pneumonia .

To determine the diagnosis of asthma, the clinician must determine that episodic symptoms of airway obstruction are present.

  • Positive family history . Asthma is a hereditary disease, and can be possibly acquired by any member of the family who has asthma within their clan.
  • Environmental factors . Seasonal changes, high pollen counts, mold, pet dander, climate changes, and air pollution are primarily associated with asthma.
  • Comorbid conditions . Comorbid conditions that may accompany asthma may include gastroeasophageal reflux, drug-induced asthma, and allergic broncopulmonary aspergillosis.

Medical Management

Immediate intervention may be necessary, because continuing and progressive dyspnea leads to increased anxiety , aggravating the situation.

  • Short-acting beta 2 – adrenergic agonists . These are the medications of choice for relief of acute symptoms and prevention of exercise-induced asthma.
  • Anticholinergics . Anticholinergics inhibit muscarinic cholinergic receptors and reduce intrinsic vagal tone of the airway.
  • Corticosteroids. Corticosteroids are most effective in alleviating symptoms, improving airway function, and decreasing peak flow variability.
  • Leukotriene modifiers. Anti Leukotrienes are potent bronchoconstrictors that also dilate blood vessels and alter permeability.
  • Immunomodulators . Prevent binding of IgE to the high affinity receptors of basophils and mast cells.

Peak Flow Meter

  • Peak flow meters. Peak flow meters measure the highest airflow during a forced expiration.
  • Daily peak flow monitoring. This is recommended for patients who meet one or more of the following criteria: have moderate or severe persistent asthma, have poor perception of changes in airflow or worsening symptoms, have unexplained response to environmental or occupational exposures, or at the discretion of the clinician or patient.
  • Function. If peak flow monitoring is used, it helps measure asthma severity and, when added to symptom monitoring, indicates the current degree of asthma control.

Nursing Management

The immediate care of patients with asthma depends on the severity of the symptoms.

Assessment of a patient with asthma includes the following:

  • Assess the patient’s respiratory status by monitoring the severity of the symptoms.
  • Assess for breath sounds.
  • Assess the patient’s peak flow.
  • Assess the level of oxygen saturation through the pulse oximeter.
  • Monitor the patient’s vital signs.

Based on the data gathered, the nursing diagnoses appropriate for the patient with asthma include:

  • Ineffective airway clearance related to increased production of mucus and bronchospasm.
  • Impaired gas exchange related to altered delivery of inspired O2.
  • Anxiety related to perceived threat of death.

Main Article:  5 Bronchial Asthma Nursing Care Plans

To achieve success in the treatment of a patient with asthma, the following goals should be applied:

  • Maintenance of airway patency.
  • Expectoration of secretions.
  • Demonstration of absence/reduction of congestion with breath sounds clear, respirations noiseless, improved oxygen exchange.
  • Verbalization of understanding of causes and therapeutic management regimen.
  • Demonstration of behaviors to improve or maintain clear airway.
  • Identification of potential complications and how to initiate appropriate preventive or corrective actions.

The nurse generally performs the following interventions:

  • Assess history. Obtain a history of allergic reactions to medications before administering medications.
  • Assess respiratory status . Assess the patient’s respiratory status by monitoring the severity of symptoms, breath sounds, peak flow, pulse oximetry, and vital signs.
  • Assess medications. Identify medications that the patient is currently taking. Administer medications as prescribed and monitor the patient’s responses to those medications; medications may include an antibiotic if the patient has an underlying respiratory infection .
  • Pharmacologic therapy. Administer medications as prescribed and monitor patient’s responses to medications.
  • Fluid therapy. Administer fluids if the patient is dehydrated.

To determine the effectiveness of the plan of care, evaluation must be performed. The following must be evaluated:

  • Expectoration or clearance of secretions.
  • Absence /reduction of congestion with breath sound clear, noiseless respirations, and improved oxygen exchange.
  • Verbalized understanding of causes and therapeutic management regimen.
  • Demonstrated behaviors to improve or maintain clear airway.
  • Identified potential complications and how to initiate appropriate preventive or corrective actions.

A major challenge is to implement basic asthma management principles at the home and community level.

  • Collaboration. The complex therapy of treating asthma at home needs collaboration between the patient and the health care provider to determine the desired outcomes and to formulate a plan to achieve those outcomes.
  • Health education. Patient teaching is a critical component of care for patients with asthma. Teach patient and family about asthma (chronic inflammatory), purpose and action of medications, triggers to avoid and how to do so, and proper inhalation technique. Instruct patient and family about peak-flow monitoring. Obtain current educational materials for the patient based on the patient’s diagnosis, causative factors, educational level, and cultural background.
  • Compliance to therapy. Nurses should emphasize adherence to the prescribed therapy, preventive measures, and the need to keep follow-up appointments with health care providers. Teach patient how to implement an action plan and how and when to seek assistance.
  • Home visits. Home visits by the nurse to assess the home environment for allergens may be indicated for patients with recurrent exacerbations.

Documentation is a necessary part of the nursing care provided, and the following data must be documented:

  • Related factors for individual client.
  • Breath sounds, presence and character of secretions, and use of accessory muscles for breathing.
  • Character of cough and sputum.
  • Respiratory rate, pulse oximetry/o2 saturation, and vital signs.
  • Plan of care and who is involved in planning.
  • Teaching plan.
  • Client’s response to interventions, teaching, and actions performed.
  • Use of respiratory devices/airway adjuncts.
  • Response to medications administered.
  • Attainment or progress towards desired outcomes.
  • Modifications to the plan of care.

Let’s reinforce what you’ve learned with this 5-item NCLEX practice quiz about Asthma. Please visit our nursing test bank for more NCLEX practice questions .

1. Histamine, a mediator that supports the inflammatory process in asthma, is secreted by:

A. Eosinophils B. Lymphocytes C. Mast cells D. Neutrophils

2. Obstruction of the airway in the patient with asthma is caused by all of the following except:

A. Thick mucus B. Swelling of bronchial membranes C. Destruction of the alveolar wall D. Contraction of muscles surrounding the bronchi

3. A commonly prescribed mast cell stabilizer used for asthma is:

A. Albuterol B. Budesonide C. Cromolyn sodium D. Theophylline

4. There are several comorbid conditions that accompany asthma but it does not include:

A. Gastroeasophageal reflux B. Drug-induced asthma C. Hypertension D. Allergic bronchopulmonary aspergillosis

5. The strongest predisposing factor for asthma is:

A. Chronic exposure to airway irritants B. Exercise C. Medications D. Allergy

Answers and Rationale

1. Answer: C. Mast cells

  • C: When the mast cells are activated, it releases several chemicals called mediators, and one of them is histamine.
  • A: Eosinophils are part of the inflammatory process in asthma.
  • B: Lymphocytes have a role in the inflammatory process in asthma.
  • D: Neutrophils are part of the inflammatory process in asthma.

2. Answer: C. Destruction of the alveolar wall

  • C: There is no destruction of the alveolar wall in asthma.
  • A: Thick mucus causes obstruction of the airway.
  • B: Swelling of the bronchial membranes causes bronchoconstriction or obstruction of the airway.
  • D: Contraction of muscles surrounding the bronchi is a cause of obstruction of the airway.

3. Answer: C. Cromolyn sodium

  • C: Cromolyn sodium is a mast stabilizer commonly prescribed for patients with asthma.
  • A: Albuterol is not a mast cell stabilizer; it is a short-acting beta 2 adrenergic agonist.
  • B: Budesonide is an inhaled corticosteroid.
  • D: Theophylline is classified as methylxanthine.

4. Answer: C. Hypertension

  • C: Hypertension is not a comorbid condition of asthma.
  • A: GERD occurs along with asthma in some patients.
  • B: Drug-induced asthma is a comorbid condition of asthma.
  • D: Allergic bronchopulmonary aspergillosis is a comorbid condition of asthma.

5. Answer: D. Allergy

  • D: Allergy is the strongest predisposing factor for asthma.
  • A: One of the causes of chronic exposure to airway irritants.
  • B: Exercise is one of the predisposing factors of asthma.
  • C: Medications can also start the development of asthma.

Posts related to Asthma:

  • Asthma and COPD NCLEX Practice Quiz 1 (50 Items)
  • Asthma and COPD NCLEX Practice Quiz 2 (50 Items)
  • 5 Bronchial Asthma Nursing Care Plans

5 thoughts on “Asthma”

Perfect for students

I would like to use some of this key information in a student case study that I’m writing but am hesitant to do so because I don’t see enough information here about how to cite the reference. I suggest you add a widget/button that provides an APA style reference citation. It would also bring alot of free advertisement. : )

Thanks I really got what I wanted

You are really great, will like to meet you one day in America

Great lecture

Leave a Comment Cancel reply

  • Join our Mailing List

Working locally in primary care and collaborating globally to improve respiratory health

Clinical case study - asthma, clinical case study - asthma, resource information.

  • Disease management
  • Search Menu
  • Advance Articles
  • Supplements
  • Author Guidelines
  • Submission Site
  • Open Access
  • Self-Archiving Policy
  • Advertising & Corporate Services
  • Advertising
  • Reprints and ePrints
  • Sponsored Supplements
  • Branded Books
  • Journals Career Network
  • About Paediatrics & Child Health
  • About The Canadian Paediatric Society
  • Editorial Board
  • Journals on Oxford Academic
  • Books on Oxford Academic

Issue Cover

Article Contents

Case 1 diagnosis: allergy bullying, clinical pearls.

  • < Previous

Case 1: A 12-year-old girl with food allergies and an acute asthma exacerbation

  • Article contents
  • Figures & tables
  • Supplementary Data

Lopamudra Das, Michelle GK Ward, Case 1: A 12-year-old girl with food allergies and an acute asthma exacerbation, Paediatrics & Child Health , Volume 19, Issue 2, February 2014, Pages 69–70, https://doi.org/10.1093/pch/19.2.69

  • Permissions Icon Permissions

A 12-year-old girl with a history of asthma presented to the emergency department with a three-day history of increased work of breathing, cough and wheezing. She reported no clear trigger for her respiratory symptoms, although she had noted some symptoms of a mild upper respiratory tract infection. With this episode, the patient had been using a short-acting bronchodilator more frequently than she had in the past, without the expected resolution of symptoms.

On the day of presentation, the patient awoke feeling ‘suffocated’ and her mother noted her lips to be blue. In the emergency department, her oxygen saturation was 85% and her respiratory rate was 40 breaths/min. She had significantly increased work of breathing and poor air entry bilaterally to both lung bases, with wheezing in the upper lung zones. She was treated with salbutamol/ipratropium and received intravenous steroids and magnesium sulfate. Her chest x-ray showed hyperinflation and no focal findings.

Her medical history revealed that she was followed by a respirologist for her asthma, had good medication adherence and had not experienced a significant exacerbation for six months. She also had a history of wheezing, dyspnea and pruritis with exposure to peanuts, chickpeas and lentils; she had been prescribed an injectible epinephrine device for this. However, her device had expired at the time of presentation. In the past, her wheezing episodes had been seasonal and related to exposure to grass and pollens; this presentation occurred during the winter. Further history revealed the probable cause of her presentation.

Although reluctant to disclose the information, our patient later revealed that she had been experiencing significant bullying at school, which was primarily related to her food allergies. Three days before her admission, classmates had smeared peanut butter on one of her schoolbooks. She developed pruritis immediately after opening the book and she started wheezing and coughing later that day. This event followed several months of being taunted with peanut products at school. The patient was experiencing low mood and reported new symptoms of anxiety related to school. The review of systems was otherwise negative, with no substance use.

The patient's asthma exacerbation resolved with conventional asthma treatment. Her pulmonary function tests were nonconcerning (forced expiratory volume in 1 s 94% and 99% of predicted) after her recovery. The trigger for her asthma exacerbation was likely multifactorial, related to exposure to the food allergen as well as the upper respiratory infection. A psychologist was consulted to assess the symptoms of anxiety and depression that had occurred as a result of the bullying. During the hospitalization, the medical team contacted the patient's school to provide education on allergy bullying, treatment of severe allergic reactions and its potential for life-threatening reactions with exposure to allergens. The medical team also recommended community resources for further education of students and staff about allergy bullying and its prevention.

Allergy bullying is a form of bullying with potentially severe medical outcomes. In recent years, it has gained increasing notoriety in schools and in the media. Population-based studies have shown that 20% to 35% of children with allergies experience bullying. In many cases (31% in one recent study [ 1 ]), this bullying is related directly to the food allergy. From a medical perspective, there are little published data regarding allergy bullying, and many health care providers may not be aware of the issue.

Allergy bullying can include teasing a child about their allergy, throwing food at a child, or even forcing them to touch or eat allergenic foods. Most episodes of allergy bullying occur at school, and can include episodes perpetrated by teachers and/or staff ( 2 ).

Allergy bullying can lead to allergic reactions, which may be mild or severe (eg, urticaria, wheezing, anaphylaxis), but may also lead to negative emotional consequences (sadness, depression) ( 2 ) and an overall decrease in quality of life measures ( 1 ). Adolescents commonly resist using medical devices, such as injectible epinephrine devices, and bullying may be a contributing factor for this ( 3 ). Attempting to conceal symptoms in a bullying situation may place children at risk for a worse outcome.

Physicians can play a key role in detecting allergy bullying and its health consequences. In many cases, children have not discussed this issue with their parents ( 1 ). Given the prevalence of bullying, its potential to lead to severe harm, including death, and the lack of awareness of this issue, clinicians should specifically ask about bullying in all children and teens with allergies. Physicians can also work with families and schools to support these children, educate their peers and school staff, and help prevent negative health outcomes from allergy bullying.

Online resources

www.anaphylaxis.ca − A national charity that aims to inform, support, educate and advocate for the needs of individuals and families living with anaphylaxis, and to support and participate in research. This website includes education modules for schools and links to local support groups throughout Canada.

www.whyriskit.ca/pages/en/live/bullying.php − A website for teenagers with food allergies; includes a segment that addresses food bullying.

www.foodallergy.org − Contains numerous resources for children and their families, including a significant discussion on bullying and ways to prevent it.

Allergy bullying is common but is often unrecognized as a factor in clinical presentations of allergic reactions.

Physicians should make a point of asking about bullying in patients with allergies and become familiar with resources for dealing with allergy bullying.

Physicians can play roles as advocates, educators and collaborators with the school system to help make the school environment safer for children with allergies who may be at risk for allergy bullying.

Google Scholar

Email alerts

Citing articles via, looking for your next opportunity.

  • About Paediatrics & Child Health
  • Recommend to Your Librarian
  • Advertising and Corporate Services

Affiliations

  • Online ISSN 1918-1485
  • Print ISSN 1205-7088
  • Copyright © 2024 Oxford University Press
  • About Oxford Academic
  • Publish journals with us
  • University press partners
  • What we publish
  • New features  
  • Open access
  • Institutional account management
  • Rights and permissions
  • Get help with access
  • Accessibility
  • Media enquiries
  • Oxford University Press
  • Oxford Languages
  • University of Oxford

Oxford University Press is a department of the University of Oxford. It furthers the University's objective of excellence in research, scholarship, and education by publishing worldwide

  • Copyright © 2024 Oxford University Press
  • Cookie settings
  • Cookie policy
  • Privacy policy
  • Legal notice

This Feature Is Available To Subscribers Only

Sign In or Create an Account

This PDF is available to Subscribers Only

For full access to this pdf, sign in to an existing account, or purchase an annual subscription.

  • Chinese, Simplified(简体中文)
  • Portuguese, Brazil

Your quote request is empty.

How to use this pediatric asthma clinical case study in your nursing course

Image

Dr. Andrew O’Malley discusses the importance of clinical cases to nursing education and shares how to use our new pediatric asthma case study to teach your nursing students about childhood asthma.

A photograph of a man called Andrew O'Malley. He is smiling at the camera. He wears a blue and white checked shirt and glasses. He is in an indoor setting.

Dr Andrew O'Malley

Case study examples are important in nursing education because they allow students to develop soft skills and apply what they’ve learned in simulated scenarios.

We spoke to Dr. Andrew O’Malley of the University of St Andrews about the new Pediatric Asthma Module he has developed in Lt, which aims to expose students to the experience, symptoms, and pathophysiology of a real case of childhood asthma.

Here, Andrew breaks down the module and shares how you can use it immediately in your course (or edit it as you see fit!).

“...the real texture in these nursing case studies [is] thinking about a distressed parent, an anxious parent. Thinking about a child who is worried that he won't get to play football with his friends. And so actually, we were actually trying to build in much more of a communication skills angle.”

Introducing nursing students to pediatric asthma

The Pediatric Asthma Module shares a real patient case with students: Zac Thayer’s experience of asthma and how it progressed throughout his childhood. It consists of 4 parts.

  • Students relate theory to practice as they complete the Airflow Lab.
  • In the pediatric asthma video Case Study, students meet Zac and his family, discover Zac’s relevant family history related to asthma, explore his tests, and learn about pediatric asthma treatment and his discharge from hospital.

An illustration of a child using an inhaler against an orange background. The text on the image reads, "NEW MODULE Pediatric Asthma" and there is an Lt logo with the tagline, "Create Share Inspire" at the bottom-right of the image.

  • In the Scenario, students are asked to perform various roles to educate Zac and his parents about his condition after his asthma diagnosis before he is discharged from hospital.
  • In the Evaluation, students summarize the module by answering questions about lung anatomy, physiology of ventilation, and pediatric asthma and its management.

Preview the Pediatric Asthma Case Study »

Each of the parts of the module are fully-editable, meaning that you can pick and choose content based on your students’ level of experience and the time constraints you are working under. Learning outcomes are provided, and can be edited according to the nursing standards in your region.

“The immersive nursing module is split into a couple of different lessons, and educators can choose what they do with it; it doesn't have to be an off-the-shelf product that you use and don't alter.”

How to use the Airflow Lab

The Airflow Lab lets students get hands-on with pulmonary function tests. By the end of the session, students will know how to:

  • perform these tests
  • measure and analyze pulmonary function variables like FEV1
  • appreciate how it feels to breathe with constricted airways, such as in asthma, and
  • compare pulmonary function variables between ‘normal’ data and ‘simulated asthma’ data.

For nursing students, who may lack a technical science background, this opportunity to get in the lab and measure pulmonary variables using a spirometer lets them link theory to practice without the added stress of dealing with a real or simulated patient.

Students are able to become more comfortable using complex equipment and working together in teams when collecting data from each other.

A screenshot an illustration in Lt showing the hardware needed for a spirometry experiment.

They also develop relevant clinical experience in using a handheld peak flow meter. Peak flow meters are widely used in clinical contexts to gauge respiratory health, and also to monitor treatment efficacy in asthma patients.

By combining this standalone lab with a relevant case study on childhood asthma, students can synthesize their learnings and understand how the mechanics and pathophysiology of asthma relates to the outcomes that they see in the case. As Andrew says:

“That's the point of medical cases, is that they give them something to hang the [theoretical] knowledge off.”

How to use the Case Study

Adding a relevant clinical case study to your curriculum means students can understand the ripple effect that a disease has throughout a family. In the case of pediatric asthma, nurses will be dealing not only with the affected child, but also with their caregivers, who may be stressed, worried, or upset. This is a valuable insight:

“The students are coming in from school often, they're really young. Their life experience is limited, which is just a function of their age. So unless they've really had a personal experience of a particular illness through their family, these nursing case study examples will give them a real insight into how certain disease processes affect a family. And I think that's often really hard to capture in a textbook or a lecture.”

A screenshot of the Lt interface showing two videos of Zac's mother discussing how pediatric asthma has impacted their lives.

Pediatric simulation scenarios make it easier for students to learn. By ‘meeting’ Zac Thayer and his real presentation of pediatric asthma, students discover his asthma history timeline and watch videos of him, his parents, and his healthcare providers, including a Respiratory Nurse Specialist. Students can answer interactive questions as they move through this respiratory case study, making sure that they reflect and record their learnings about asthma in children.

A drag-and-drop question in Lt in which students are asked to label the image to review the structure of a bronchus.

“There's lots of interactives in there for the students to get personalized, instant feedback as they go through.”

Linking back to the Airflow Lab, students explore typical peak flow test results for an asthmatic child like Zac. If you’ve chosen to provide students with the lab first, then this step lets them situate their theoretical knowledge within a real case.

Providing each student with the same medical case also improves equitable access; students will see different patients in practice, but this is an opportunity for them to learn and question the same case, in a safe space.

A screenshot of the Lt interface showing an illustration of a chart showing peak flow results for a five-year-old patient with respiratory issues.

Students are also encouraged to think about trends within the data they are seeing, and to consider what external factors might be at play to influence the peak flow values they observe. In this pediatric asthma case, this might be pollen in the air outdoors, indoor air pollution during winter, or more dust mites during autumn.

When a young child like Zac is admitted to the hospital, part of a nurse's role is to monitor them for signs of respiratory distress. Additionally, nurses must be able to describe these warning signs to parents and caregivers to help them identify signs of respiratory distress once the child has been discharged from the hospital. Thus, we’ve made sure that students learn about common signs of respiratory distress in this case study, including:

  • respiratory accessory muscle use
  • respiratory retractions

Finally, students learn about discharging the patient, including patient education (discussing certain topics with the patient and their caregivers before they leave the hospital). In the case of pediatric asthma, students might discuss:

  • how to reduce asthma triggers
  • how to use inhalers, spacers, and nebulizers properly
  • how to keep a peak flow diary, and
  • how to develop an Asthma Action Plan.

“...when we get to the end of the case, the safe discharge or the observation of the patient is actually about, “Well, how do you use your asthma inhaler? What do you do if you begin to feel you're having an asthma attack? How do you know if your asthma is getting worse? How are we going to stop you from having an attack next time?” But it's not, “I'm going to prescribe you X milligrams of salbutamol, and we'll up it in 3 days,” - that's the doctor's job.”

How to use the Scenario

In the asthma case Scenario, students are able to practice their nursing skills in a ward-like setting. This lesson is a guide during a simulated clinical experience under educator supervision.

“And then we've got the Patient Education section, which actually is as important as the story. It focuses on the knowledge and understanding and skills that the patient needs to have with them that will allow them to be safe and to manage their condition. So for this asthma case it might be being safe while going to school, or playing with your friends.”

Students take on a variety of roles in this simulation, including:

  • Registered nurse: The student needs to  explain what it means for Zac to have asthma and how the two types of medication provided work to alleviate and prevent symptoms. The student also completes an Asthma Action Plan alongside the parents, and explains what needs to happen in each instance of Zac's condition worsening. It is important to correctly assess how much the parents comprehend.
  • Student nurse: The student needs to teach Zac's parents how to help Zac correctly complete a peak flow meter test, and show them how to record the results in a peak flow diary. The student should explain how certain triggers might make his asthma worse, which can explain the fluctuations in peak from day-to-day and season-to-season. It is important to correctly assess how much the parents comprehend.
  • Zac's parents: The student acts as a parent, who is very worried about future asthma attacks for Zac. They  are unsure when it is appropriate to stay at home, or when to call an ambulance. They are emotionally and mentally exhausted after Zac's hospital stay and very eager to return home for a family meal. Their attention span feels limited after the stress of the past few days.

This scenario builds empathy in the students as they learn how to interact with Zac’s parents, and act as the parents themselves. Patient scenario examples help to build this important skill for nurses, who will be working in stressful environments, and with people who may have heightened emotions.

“It really ties together the basic anatomy, physiology, pathology, pharmacology, all the basic science, with the real outcome that the patient wants, for them to go out and enjoy their time.”

How to use the Evaluation

The Evaluation lesson is intended to be used at the end of the module, and tests essential student knowledge. Depending on your course aims, this could be a summative assessment, or you could choose to assign it to students as a formative assessment to work through in their own time.

“There's a bit of storytelling, a lot of patient safety, a lot of nurse management and then, from the student perspective, it really, clearly signals to the student whether they've understood what they're working with. We've tried to avoid big blocks of text and didactic heavy reading.”

A screenshot of the Lt interface showing a drag-and-drop question in Lt in which students are asked to label the structures of the bronchial tree.

If you do choose to use the Evaluation in your course, the ready-made feedback supports students to take control of their own learning, which is an important skill to learn in a case-based curriculum.

“The inherent ability to create interactive, immediate feedback within Lt is really good. So most of it actually isn't us teaching the students about this thing, it's the student watching a video, reflecting on it, making some notes, answering a question, getting good feedback.”

We recommend that you try to tie your theoretical lectures and labs as closely as possible to clinical cases in your nursing course curriculum.

Rather than a traditional curriculum, which might expose students to the physiology of breathing in year 1, pediatric asthma pathophysiology in year 2, and patient interactions in year 3, a blended curriculum means that students can apply their theoretical knowledge immediately, and appreciate the relevance of what they are learning for their future careers.

In Andrew’s teaching, each week of his course starts with a case, which kicks off a week of theoretical and clinically-situated learning. Each week is a “mini medical degree” and students appreciate the fact that their learning is integrated.

“Cases are a trigger for learning at the beginning of the week. It inspires the student to connect with the content. Giving a student a real experience with a real patient with asthma means that all that learning they've done about bronchodilation and smooth muscle receptors is extremely relevant, and it makes it much more likely that they'll retain that information.”

Providing your nursing students with relevant case studies has many benefits. Students develop an appreciation for the complex factors at play within families, how conditions are treated, what to look for when observing patients in the hospital, and how to approach patient education.

By starting with a ready-made Case Module, you can provide these benefits to your students while saving time - and their editable nature means that your cases will always be relevant. You can also branch out by pulling from our 30+ Patient Resource Packs on a range of relevant medical conditions.

“It's all about relevance. The more your content aligns with other content, it’s a compound effect, they all become much more valuable.”

Contact us to learn more about the Immersive Nursing Collection in Lt »

Dr. Andrew O'Malley

Dr. Andrew O’Malley Senior Lecturer School of Medicine University of St Andrews    Andrew graduated with a PhD in Anatomy & Human Identification from the University of Dundee in 2013. He has worked as a Forensic Consultant with the United Nations and has a particular interest in medical education.

He is currently the ScotGEM Deputy Program Director at the University of St Andrews. ScotGEM is a unique 4-year program that develops doctors for rural practice in Scotland.

Are you interested in using Lt to deliver your course? Try Lt now – for free!

Nursing Care Plan (NCP) for Asthma

Watch More! Unlock the full videos with a FREE trial

Included In This Lesson

Study tools.

Access More! View the full outline and transcript with a FREE trial

Introduction to Nursing Care Plan (NCP) for Asthma

Definition of asthma.

Asthma is a chronic respiratory condition characterized by bronchoconstriction, inflammation of airways, and increased mucus production. These factors collectively narrow the air passages, reducing the ability to bring air into the alveoli, and subsequently impairing the exchange of oxygen and carbon dioxide.

Asthma can lead to significant respiratory distress, hypoxia, and decreased overall quality of life. Understanding the pathophysiology, etiology, and appropriate nursing interventions for asthma is crucial for nurses in order to provide optimal care and support for individuals affected by this condition.

Triggers for Asthma

While asthma is not always preventable, identifying and managing its triggers can significantly improve a patient’s quality of life and reduce the frequency and severity of asthma attacks.

A helpful mnemonic for remembering common asthma triggers is:

A – allergens (seasonal, animal, food)

One of the most well-known triggers for asthma is exposure to allergens. Allergens are substances that can stimulate an allergic reaction in some individuals, leading to airway inflammation and bronchoconstriction. Seasonal allergens, such as pollen from trees, grasses, and weeds, are notorious culprits, often causing “seasonal allergies” or allergic rhinitis. Additionally, allergens from animals, particularly pet dander, saliva, and urine, can trigger asthma symptoms. Foods can also be asthma triggers, with common culprits including peanuts, tree nuts, shellfish, and dairy products.

S – sports or smoking

Physical activity, particularly in cold or dry environments, can trigger asthma symptoms in some individuals. This phenomenon, known as exercise-induced bronchoconstriction (EIB) or exercise-induced asthma (EIA), occurs due to the loss of heat and moisture from the airways during heavy breathing. Smoking and exposure to secondhand smoke are well-established asthma triggers. Cigarette smoke contains numerous irritants that can worsen airway inflammation and lead to more severe asthma symptoms.

T – temperatures (change in season, cold air)

Cold air, in particular, is known to induce bronchoconstriction in individuals with sensitive airways. This is often exacerbated during the winter months when people spend more time indoors with indoor heating systems that can dry out the air.

H – hazards (chemicals)

Common indoor irritants include household cleaning products, perfumes, and strong odors. Outdoor air pollution, including particulate matter and ozone, can also be asthma triggers. Occupational exposures to specific chemicals or allergens in the workplace can lead to occupational asthma, a form of asthma that develops or worsens due to workplace conditions.

M – microbe (infection)

Respiratory infections, particularly viral infections like the common cold and influenza, are well-known asthma triggers. Infections can cause airway inflammation and increase mucus production, exacerbating asthma symptoms.

A – anxiety

Emotional factors, including anxiety and stress, can contribute to asthma symptoms. When a person experiences anxiety or heightened stress levels, their breathing pattern may change, leading to shallower and faster breathing. This altered breathing pattern can trigger or worsen asthma symptoms, emphasizing the mind-body connection in asthma management.

Pathophysiology For Asthma

Asthma’s pathophysiology revolves around three primary mechanisms: bronchoconstriction, inflammation, and increased mucus production. These processes collectively contribute to the narrowing of air passages, reducing the efficiency of oxygen exchange and causing difficulties in breathing.

Bronchoconstriction: In asthma, the smooth muscles surrounding the airways undergo spasms or contractions, leading to bronchoconstriction. This narrows the air passages and restricts the flow of air into the alveoli—the tiny air sacs in the lungs where oxygen is absorbed into the bloodstream.

Bronchoconstriction significantly reduces the patient’s ability to inhale adequate amounts of oxygen.

Inflammation: Chronic inflammation of the airways is a hallmark of asthma. In response to irritants or triggers, such as dust, pollen, smoke, or infection, the airway lining becomes inflamed. This inflammation further narrows the air passages and leads to the production of excess mucus. Inflammation can also make the airway walls thicker, increasing the resistance to airflow.

Increased Mucus Production: The inflammatory response in asthma prompts the airway epithelial cells to produce excessive mucus. This mucus accumulates within the airways, further obstructing airflow.

The combination of bronchoconstriction, inflammation, and mucus production creates a hostile environment for efficient gas exchange.

Etiology For Asthma

Asthma can have various etiological factors, including genetic, environmental, and trigger-related causes. Understanding these factors is essential for effective asthma management.

Genetic Factors for Asthma: Genetic predisposition plays a significant role in asthma development. Individuals with a family history of asthma are more likely to develop the condition themselves. Specific genetic markers and variations are associated with an increased susceptibility to asthma.

Environmental Factors Leading to Asthma: Environmental factors can also contribute to asthma. Exposure to allergens and irritants such as pollen, dust mites, smoke, or air pollution can trigger asthma symptoms in susceptible individuals. Additionally, indoor air quality, humidity levels, and exposure to secondhand smoke can impact asthma severity.

Triggers that lead to asthma: Asthma can be triggered by a wide range of factors, including respiratory infections, exercise, and allergies. Respiratory infections, such as viral or bacterial infections, can exacerbate asthma symptoms by increasing airway inflammation. Physical activity, while generally beneficial, can trigger exercise-induced bronchoconstriction in some individuals with asthma. Allergies to substances like pollen, animal dander, or certain foods can also provoke asthma attacks.

Desired Outcome For Asthma

The primary goal in managing asthma is to achieve a desired outcome that includes:

  • Decreased Work of Breathing: Patients should experience reduced respiratory distress, making breathing easier and less labored.
  • Adequate Ventilation and Oxygenation: Ensuring that patients can inhale and exhale effectively, facilitating sufficient oxygen intake, and the removal of carbon dioxide.
  • Perfusion of Oxygen-Rich Blood to Tissues: Optimizing oxygen saturation in the bloodstream, ensuring that vital organs and tissues receive an adequate supply of oxygen.

Asthma Nursing Care Plan (NCP)

To achieve the desired outcome for asthma management, nurses play a crucial role in assessing patients, implementing interventions, and providing education and support. The following nursing care plan outlines the essential elements of care for individuals with asthma.

Subjective Data:

Definition of subjective data: information provided by individuals about their symptoms, feelings, and medical history, relying on their personal descriptions and experiences, rather than objective measurements or observations.

  • “I can’t breathe”
  • Chest Pressure
  • Chest Tightness
  • Reported Cough
  • The patient mentions any recent triggers or exacerbating factors (e.g., exposure to allergens, respiratory infections).

Objective Data:

Definition of Objective Data: measurable and observable information collected through physical examinations, diagnostic tests, and direct observations, providing quantifiable and factual details about a patient’s physical condition, such as vital signs, laboratory results, physical findings, and imaging findings.

  • Observed Cough
  • Pursed lip breathing
  • Low pulse oximetry (<90%)
  • Blue lips/fingers
  • Tachypnea – abnormally rapid breathing, with a respiratory rate exceeding the normal range for a person’s age and condition.
  • Tripod position – a person with respiratory distress leans forward and supports themselves with their hands on their knees or another surface to assist with breathing by allowing improved chest expansion.

Nursing Interventions and Rationales For Asthma

Check pulse oximetry:.

Apply oxygen if O2 saturation is less than 90%, starting at 2 liters nasal cannula (2L NC).

Rationale: Monitoring oxygen saturation levels is critical to assess the patient’s oxygenation status. Applying supplemental oxygen helps alleviate hypoxia and ensures the patient receives adequate oxygen. Oxygen therapy is a safe and effective intervention, but it should be administered cautiously and adjusted as needed.

Auscultate Lung Sounds:

If wheezing is present, consider administering a bronchodilator. If crackles or rhonchi are heard, pneumonia may be suspected, and suctioning may be necessary.

Rationale: Auscultating lung sounds provide valuable information about the patient’s respiratory status. Wheezing indicates airway constriction, while crackles or rhonchi may suggest airway secretions or infection. Prompt intervention is essential to manage these issues effectively.

Educate About Triggers:

Ensure the patient’s environment is free from asthma triggers, such as dust, pollen, or smoke.

Rationale: Identifying and eliminating triggers is a fundamental aspect of asthma management. Educating patients about their specific triggers empowers them to take proactive measures to minimize exposure, thus reducing the risk of asthma exacerbations.

Positioning the Patient in an Upright Position:

Rationale: Placing the patient in an upright position helps open the lung bases and airway, making it easier for the patient to breathe. This position can alleviate respiratory distress and improve oxygenation.

Have the Patient Perform a Peak Flow Meter:

Rationale: Peak flow meters are valuable tools for monitoring asthma severity. They measure the patient’s ability to exhale air forcefully, providing an objective assessment of lung function. A decrease in peak flow readings may indicate worsening asthma, prompting the need for intervention.

Breathing Treatments and Medication Therapy:

  • Beta-Agonists (e.g., albuterol) work as bronchodilators.
  • Anticholinergics (e.g., Ipratropium) relax bronchospasm.
  • Corticosteroids (e.g., Fluticasone) act as anti-inflammatories.

Rationale: Medications play a pivotal role in asthma management. Bronchodilators help relieve bronchoconstriction, while anti-inflammatories reduce airway inflammation. Administering the appropriate medications as prescribed is essential for symptom control and improved lung function.

Be Prepared with an Airway Cart for Critical Cases:

Rationale: In severe asthma aggravations, especially in pediatric patients or those who have been struggling to breathe intensely for an extended period, it is important to be prepared with an airway cart. This proactive approach ensures patient safety and allows for prompt intervention.

It is also important to involve respiratory therapists promptly, as they possess expertise in managing complex respiratory cases. Securing the airway should not be delayed, as an impending airway closure can lead to life-threatening situations. In certain instances, sedation and intubation may be necessary to address respiratory acidosis or alkalosis and stabilize the patient’s condition.

***Safety*** You do not want to wait until the impending airway closure happens to try to secure their airway. Sometimes the patient will be sedated and intubated to try to correct any respiratory acidosis or alkalosis.

Linchpin On Nursing Care Plan (NCP) For Asthma

Asthma is a multifaceted respiratory condition with complex pathophysiological mechanisms and various contributing factors. Effective management of asthma involves a comprehensive approach, encompassing a thorough understanding of its pathophysiology, identification of triggers, and the implementation of tailored nursing interventions.

By combining both subjective and objective data, you can provide patient-centered care that addresses the unique needs of individuals living with asthma. This comprehensive nursing care plan outlines essential interventions that aim to reduce respiratory distress, enhance oxygenation, and promote overall well-being.

Through a collaborative effort involving healthcare providers, patients, and their families, individuals with asthma can achieve improved clinical outcomes, increased comfort, and a higher quality of life. By adhering to evidence-based guidelines and tailoring care to each patient’s specific needs, nurses play a crucial role in helping individuals with asthma manage their condition effectively and live healthier lives.

  • http://www.lung.org
  • http://www.lung.org/lung-health-and-diseases/lung-disease-lookup/asthma/living-with-asthma/managing-asthma/measuring-your-peak-flow-rate.html
  • http://www.lung.org/lung-health-and-diseases/lung-disease-lookup/asthma/asthma-symptoms-causes-risk-factors/what-causes-asthma.html
  • https://www.uptodate.com/contents/management-of-acute-exacerbations-of-asthma-in-adults

View the FULL Outline

When you start a FREE trial you gain access to the full outline as well as:

  • SIMCLEX (NCLEX Simulator)
  • 6,500+ Practice NCLEX Questions
  • 2,000+ HD Videos
  • 300+ Nursing Cheatsheets

“Would suggest to all nursing students . . . Guaranteed to ease the stress!”

View the FULL Transcript

Nursing care plans.

Jon Haws

How do I write a Nursing Care Plan? Why and how do we even use Nursing Care Plans ? This course is going to expand on that for you and show you the most effective way to write a Nursing Care Plan and how to use Nursing Care Plans in the clinical setting . PLUS, we are going to give you examples of Nursing Care Plans for all the major body systems and some of the most common disease processes. When you complete this course, you will be able to write and implement powerful and effective Nursing Care Plans.

0 – Nursing Care Plans Course Introduction

1 – understanding nursing care plans.

  • 16 Questions

Cardiovascular (Cardiac, CVD) Care Plans

  • 5 Questions
  • 6 Questions
  • 10 Questions
  • 3 Questions
  • 8 Questions
  • 1 Questions
  • 4 Questions

Eyes, Ears, Nose, Throat (EENT) Care Plans

  • 7 Questions

Gastrointestinal (GI) Care Plans

  • 2 Questions

Genitourinary (Renal) (Kidney) (Nephrotic) Care Plans

Hematology (blood, labs), oncology (cancer) & immunology (immunity) care plans, integumentary (skin) care plans, mental health care plans, metabolic & endocrine care plans.

  • 9 Questions

Neurological Care Plans

Obstetrics (ob) & pediatrics (peds) care plans, respiratory care plans.

  • 11 Questions

Sepsis (Septic) & Shock Care Plans

Musculoskeletal and skeletal (osteo) (bones) care plans.

IMAGES

  1. Asthma Nursing Care Management and Study Guide

    nursing case study examples asthma

  2. SOAP Note for an Asthmatic Patient

    nursing case study examples asthma

  3. Sample Nursing Case Study

    nursing case study examples asthma

  4. Case Study of Asthma

    nursing case study examples asthma

  5. Case Study On Asthma

    nursing case study examples asthma

  6. PPT

    nursing case study examples asthma

VIDEO

  1. Paediatrics Long Case

  2. #CASE STUDY ON ASTHMA #NURSING #EDUCATION

  3. CLINICAL PRESENTATION / CASE STUDY on Asthma

  4. nursing care plan on asthma medical surgical nursing 2 nd year #youtube #healthcare #creater

  5. ASTHMA (Bronchialasthma) Care Plan #nursing #gnm #nursingcareplan #nursing_criteria #nursingstudent

  6. Respiratory Disorders in urdo

COMMENTS

  1. Case Study: 60-Year-Old Female Presenting With Shortness of Breath

    Case Presentation. The patient is a 60-year-old white female presenting to the emergency department with acute onset shortness of breath. Symptoms began approximately 2 days before and had progressively worsened with no associated, aggravating, or relieving factors noted. She had similar symptoms approximately 1 year ago with an acute, chronic ...

  2. Case Study: Managing Severe Asthma in an Adult

    The majority of adverse effects occurred within 1 day of the procedure and resolved within 7 days. 6. In this study, bronchial thermoplasty was found to significantly improve quality of life, as ...

  3. Nursing Case Study for Pediatric Asthma

    Hey everyone. My name is Abby. We're going to cover a case study today about pediatric asthma. Let's get started. Anthony is a six year old male patient brought to the ER with a history of asthma. Since he came home from the NICU as an infant, he has lived with his parents, Bob and Josh, who adopted him after fostering him from the age of four ...

  4. A woman with asthma: a whole systems approach to supporting self

    A number of studies have demonstrated the challenges for primary care physicians in providing ongoing support for people with asthma. 31,48,49 In some countries, nurses and other allied health ...

  5. Nursing Diagnosis for Asthma: 8 Nursing Care Plans

    Assess for factors related to the cause of asthma: Swelling and spasm of the bronchial tubes in response to inhaled irritants, infection, drugs, allergies, or infection. Bronchial secretions. Inability to mobilize and spontaneously expel secretions. Bronchospasms. Increased pulmonary secretions.

  6. Asthma Nursing Care Management and Study Guide

    Nursing Diagnosis. Based on the data gathered, the nursing diagnoses appropriate for the patient with asthma include: Ineffective airway clearance related to increased production of mucus and bronchospasm. Impaired gas exchange related to altered delivery of inspired O2. Anxiety related to perceived threat of death.

  7. Clinical case study

    Clinical case study - asthma . 2019 . Clinical Case Study - Asthma. pdf. Clinical Case Study - Asthma. 6.34 MB. Resource information. Respiratory conditions. Asthma; Respiratory topics. Disease management; Diagnosis; Type of resource. Presentation . Author(s) Jaime Correia de Sousa Ioanna Tsiligianni Miguel Román Rodriguez

  8. PDF An interprofessional case study

    At the end of this presentation students will be able to: Give a definition of asthma. Discuss what could cause asthma. Identify four triggers of asthma. Describe symptoms of an asthma flare-up. Explain three asthma treatment strategies. Develop an interprofessional plan of care for an asthma sufferer.

  9. PDF Learning the Asthma Guidelines by Case Studies

    1. Tiotropium Bromide can be added in place of a LABA to a moderate to high dose of ICS if the patient is not controlled or to a ICS/LABA combination for poor control. 2. Ipratropium bromide can be used in the ED when albuterol use in maximized and patient still has symptoms.

  10. Case 1: A 12-year-old girl with food allergies and an acute asthma

    In recent years, it has gained increasing notoriety in schools and in the media. Population-based studies have shown that 20% to 35% of children with allergies experience bullying. In many cases (31% in one recent study ), this bullying is related directly to the food allergy. From a medical perspective, there are little published data ...

  11. Integration of holistic nursing care in the treatment of asthma: A case

    "Mary," a 39-year-old woman, was in acute respiratory distress when she arrived at the emergency department. She was gasping and could speak only in 2- or 3-word sentences. She was extremely dyspneic, anxious, hyperventilating, and emotionally drained, with tears streaming down her face. She had a history of asthma and was experiencing an exacerbation of this condition. Mary's skin was ...

  12. How to use this pediatric asthma clinical case study in your nursing

    Introducing nursing students to pediatric asthma. The Pediatric Asthma Module shares a real patient case with students: Zac Thayer's experience of asthma and how it progressed throughout his childhood. It consists of 4 parts. Students relate theory to practice as they complete the Airflow Lab. In the pediatric asthma video Case Study ...

  13. PDF Asthma Right Care case studies

    Primary Care Respiratory Update. The uncomfortable outputs of the NRAD1 report - why overuse of SABA can be indicative of poor asthma control. Understanding the signs and symptoms of asthma and early recognition of increasing symptoms - effective self-monitor-ing. Use of the Asthma Control Test and understanding of the scores.

  14. Nursing Care Plan (NCP) for Asthma

    Linchpin On Nursing Care Plan (NCP) For Asthma. Asthma is a multifaceted respiratory condition with complex pathophysiological mechanisms and various contributing factors. Effective management of asthma involves a comprehensive approach, encompassing a thorough understanding of its pathophysiology, identification of triggers, and the ...

  15. Pediatric asthma case management: A review of evidence and an

    Asthma is a complex disease that involves physiological, environmental, and psychosocial factors. This paper reviews childhood asthma case management by social service professionals, lay health workers, and nurses, and it presents a new randomized controlled study using nurse case management in a local community coalition. Evidence suggests the common factor for success involves case managers ...